Equine End Sem 1 Exam Flashcards

1
Q

Diagnosis of equine coronavirus is best made using which diagnostic test?
a) Faecal PCR
b) Faecal ELISA
c) Serum ELISA

A

a) Faecal PCR

How well did you know this?
1
Not at all
2
3
4
5
Perfectly
2
Q

What is the main risk factor for Clostridium difficile diarrhoea?
a) Antimicrobial administration
b) NSAID administration
c) Anthelmintic administration

A

a) Antimicrobial administration

3rd gen Cephalosporins (e.g. Ceftiofur) antimicrobial linked to Clostridium difficile

How well did you know this?
1
Not at all
2
3
4
5
Perfectly
3
Q

Serial faecal samples are recommended when testing for which infectious agent?
a) Clostridium perfringens
b) Salmonella species

A

b) Salmonella species

How well did you know this?
1
Not at all
2
3
4
5
Perfectly
4
Q

A 5-month old foal is presented with severe respiratory distress, bilateral purulent nasal discharge and marked swelling in the thoratlatch area. The foal has a markedly increased inspiratory effort with a loud inspiratory noise which is becoming progressively louder. The foal is agitated and becomes very distressed while examining. The most appropriate course of action at this time is to:
a) Place a rebreathing bag over the foal’s nose and auscultate the thorax
b) Perform an emergency tracheostomy to relieve airway obstruction
c) Sedate the foal and obtain lateral radiographs of the thorax
d) Obtain swabs of the exudate and submit for culture and sensitivity testing
e) Administer gentamicin and penicillin and monitor the foal’s response

A

b) Perform an emergency tracheostomy to relieve airway obstruction

How well did you know this?
1
Not at all
2
3
4
5
Perfectly
5
Q

A horse is presented with trauma due to collision with a fence during training. On physical examination the horse has severe respiratory distress and there is a dorsally located wound over the 6th and 7th ribs of the left hemithorax. The primary aims of emergency management in this case are to:
a) Prevent air moving into the chest via the wound, and early re-expansion of the lungs
b) Control haemorrhage with pressure and achieve rapid primary closure of the wound
c) Control pain with analgesics and administer high doses of antibiotics intravenously
d) Administer general anaesthesia and surgically explore and debride the thoracic wound
e) Administer a balanced electrolyte solution IV and bronchodilator therapy

A

a) Prevent air moving into the chest via the wound, and early re-expansion of the lungs

How well did you know this?
1
Not at all
2
3
4
5
Perfectly
6
Q

A 3 year old Standardbred horse is presented with a history of excessive respiratory noise during exercise. Following examination determined that the noise is due to excessive nasal flutter. This noise is best described as:
a) Low pitched whistling or snoring that is continuous, but loudest during expiration
b) Low pitched whistling or snoring that is continuous but loudest during inspiration
c) High pitched whistling or snoring that is intermittend and loudest during inspiration
d) A choking or gagging sound that is loudest during inspiration when the horse is at rest
e) A intermittent roaring sound that is loudest during inspiration, when the horse is exercising

A

a) Low pitched whistling or snoring that is continuous, but loudest during expiration

How well did you know this?
1
Not at all
2
3
4
5
Perfectly
7
Q

A 2 year old Thoroughbred horse is presented with history of exercise intolerance and abnormal respiratory noise during exercise. During physical examination it is noted that airflow from the left nostril is markedly decreased compared to airflow from the right nostril. The most appropriate approach to this case is to:
a) Collect and submit blood and serum for routine haematology and biochemical evaluation
b) Obtain a tracheal aspirate and submit for cytology, bacterial culture and sensitivity testing
c) Perform both endoscopic and radiographic examinations of the upper respiratory tract
d) Perform a thoracocentesis and a radiographic examination of the lower respiratory tract
e) Perform centesis of the frontal and maxillary sinuses and submit aspirates for cytology

A

c) Perform both endoscopic and radiographic examinations of the upper respiratory tract

How well did you know this?
1
Not at all
2
3
4
5
Perfectly
8
Q

A 40kg dysmature foal was born approx. 10h ago. The feeding plan for this foal is based on a 6% body weight per day and feedings every 2 hours via NG tube.
How much milk would you administer to this foal per feed?
a) 2.4L
b) 2400mL
c) 2000mL
d) 240mL
e) 200mL

A

e) 200mL

0.06 (6%) x 40 (BW in kg) = 2.4
2.4 / 12 feedings per day if every 2 hours = 0.2L = 200mL

How well did you know this?
1
Not at all
2
3
4
5
Perfectly
9
Q

A 5 year old horse presents with bilateral epistaxis, approx 2 weeks after having pyrexia, nasal discharge and a cough for 3 days duration. Physical exam reveals: HR 44bpm, RR 16bpm, rectal temp 37.8, CRT 2 sec and petechiation of the oral mucosa. Prolonged bleeding after venepuncture is noticed. CBC reveals: PCV 30%, TS 56g/L, total leukocyte count 5.4 x 10^9/L and thrombocyte count 26 x 10^9/L. There were no platelet clumps seen on blood smear and clotting times (PT and aPTT) were within normal reference ranges. Which of the following diagnoses is most likely?
a) Myeloproliferative disease
b) Immune-mediated thrombocytopenia
c) Disseminated intravascular coagulopathy (DIC)
d) Chronic haemorrhage

A

c) Disseminated intravascular coagulopathy (DIC)

How well did you know this?
1
Not at all
2
3
4
5
Perfectly
10
Q

Which of the following options is an unacceptable method of euthanasia when used on its own, because it will induce loss of muscle tone prior to loss of conciousness?
a) Intracardiac barbiturate injection
b) Succinylcholine IM injection
c) Intrathecal lignocaine
d) Exsanguination
e) Barbiturate IV injection

A

d) Exsanguination

How well did you know this?
1
Not at all
2
3
4
5
Perfectly
11
Q

Which of the following statements regarding enteroliths in horses is correct?
a) Most commonly found in the right dorsal colon, Lucerne hay is a predisposing factor and horses often present with intermittent colic
b) Most commonly found in the right dorsal colon, grass hay is a predisposing factor and horses often present with intermittent colic
c) Most commonly found in the left dorsal colon, grass hay is a predisposing factor and horses often present with intermittent colic
d) Most commonly found in the left dorsal colon, Lucerne hay is a predisposing factor and horses often present with intermittent colic

A

a) Most commonly found in the right dorsal colon, Lucerne hay is a predisposing factor and horses often present with intermittent colic

How well did you know this?
1
Not at all
2
3
4
5
Perfectly
12
Q

A 2 day old foal is straining, tail flagging and occasionally in dorsal recumbency. The foal is not nursing but has passed meconium with yellow faeces present on the perineum. Which of the following is the MOST appropriate immediate plan for this foal?
a) Complete physical examination, blood work including PCV/TP/lactate/glucose, faecal egg count, digital rectal examination
b) Complete physical exam, blood work including PCV/TP/lactate/glucose, replace fluid deficits, enteral mare’s milk
c) Complete physical exam, blood work including PCV/TP/lactate/glucose, faecal egg count, replace fluid deficits, enteral mare’s milk
d) Complete physical exam, blood work including PCV/TP/lactate/glucose, abdominal ultrasound, abdominal fluids, replace fluid deficits

A

c) Complete physical exam, blood work including PCV/TP/lactate/glucose, faecal egg count, replace fluid deficits, enteral mare’s milk

How well did you know this?
1
Not at all
2
3
4
5
Perfectly
13
Q

Fill in the blanks of the following statement: Ideally euthanasia methods should result in rapid loss of (blank), followed by (blank) or (blank) arrest and the subsequent loss of (blank) function

A

Ideally euthanasia methods should result in rapid loss of CONCIOUSNESS, followed by RESPIRATORY or CARDIAC arrest and the subsequent loss of NEUROLOGICAL function

How well did you know this?
1
Not at all
2
3
4
5
Perfectly
14
Q

One of the hallmark radiographic signs of pleural effusion is that the lung is retracted from the thoracic wall and the material outside of the lung is more lucent than the lung itself. True / False?

A

True

How well did you know this?
1
Not at all
2
3
4
5
Perfectly
15
Q

You are scheduled to perform a general anaesthesia on a horse for field castration. Which of the following anaesthesia maintenance techniques would be LEAST appropriate?
a) An IV infusion of midazolam, ketamine & medetomidine for up to 40-60 mins
b) An IV infusion of guaifenesin, ketamine & xylazine for up to 40-60 mins
c) Inhalation anaesthesia with Isoflurane & O2 for up to 40-60 mins
d) Additional doses of ketamine & xylazine (1/2 to 1/4 of original premedication induction doses) given every 10-15 mins for up to 40-60 mins

A

c) Inhalation anaesthesia with Isoflurane & O2 for up to 40-60 mins

How well did you know this?
1
Not at all
2
3
4
5
Perfectly
16
Q

Identify the structures labelled A & B on the ultrasound image.

A

A = Liver
B = Duodenum

How well did you know this?
1
Not at all
2
3
4
5
Perfectly
17
Q

Image of cytology - mixed bacteria & plant material (arrow) surrounded by inflammatory cells (consisting of degenerate neutrophils with phagocytised bacteria).
Which of the following diagnoses is most likely?
a) Intestinal rupture
b) Enterocentesis
c) Infectious peritonitis
d) Small intestinal strangulating lesion

A

a) Intestinal rupture

How well did you know this?
1
Not at all
2
3
4
5
Perfectly
18
Q

A 14 year old Quarter horse gelding presents with lethargy and a 2 day history of decreased appetite. The initial blood work and a picture of the blood sample are shown below.
Total bilirubin massively increased.
a) Likely due to anorexia leading to an increase in unconjugated bilirubin
b) Likely due to anorexia leading to an increase in conjugated bilirubin
c) Likely due to haemolysis leading to an increase in unconjugated bilirubin
d) Likely due to haemolysis leading to an increase in conjugated bilirubin

A

a) Likely due to anorexia leading to an increase in unconjugated bilirubin

How well did you know this?
1
Not at all
2
3
4
5
Perfectly
19
Q

Which of the following dental conditions is most frequently associated with a marked limitation in lateral excursion of the mandible?
a) Chisel mouth
b) Shear mouth
c) Wave mouth
d) Step mouth

A

b) Shear mouth

How well did you know this?
1
Not at all
2
3
4
5
Perfectly
20
Q

Which of the following options describes the chronological order & initial treatment of an acutely burned horse best?
a) Cooling of affected area for 20 min and ice boot application
b) Cooling of affected area for 10 min and IV clenbuterol administration
c) Cooling of affected area for 5 min and IV fluid therapy
d) Cooling of affected area for 15 min and systemic analgesia

A

a) Cooling of affected area for 20 min and ice boot application

How well did you know this?
1
Not at all
2
3
4
5
Perfectly
21
Q

Pleuropneumonia and pleural effusion cases often require thoracic drainage. Which of the following statements regarding pleural effusion and thoracic drainage in horses is CORRECT?
a) The best way to determine if the chest drain can be removed is to observe the amount of fluid flowing through the drain
b) It is important to perform an ultrasound examination in order to estimate continued pleural effusion and accumulation and timing of drain removal
c) In horses it is only necessary to perform thoracic drainage on one side of the thorax
d) Pleural effusion should not be drained as it is often a septic effusion with fibrin build up which can result in draining tract cellulitis

A

b) It is important to perform an ultrasound examination in order to estimate continued pleural effusion and accumulation and timing of drain removal

How well did you know this?
1
Not at all
2
3
4
5
Perfectly
22
Q

Foals should ideally have their serum IgG concentration assessed at (blank) age to allow prompt treatment with (blank) or (blank) if failure of transfer of passive immunity has been detected.
a) 12-24 hours: IV plasma, systemic antimicrobials
b) >36 hours: IV plasma, systemic antimicrobials
c) >36 hours: stored per oral colostrum, IV plasma
d) 12-24 hours: stored per oral colostrum, IV plasma

A

d) 12-24 hours: stored per oral colostrum, IV plasma

How well did you know this?
1
Not at all
2
3
4
5
Perfectly
23
Q

The pleural space always contains fluid to a degree that is visible on a radiograph to reduce lung friction against the body wall during respiration. True / False?

A

False

How well did you know this?
1
Not at all
2
3
4
5
Perfectly
24
Q

Tube with red (sanguinous) fluid. Which of the following diagnoses is most likely?
a) Strangulating pedunculated lipoma
b) Small intestinal volvulus
c) Ascarid impaction
d) Ileal impaction
e) Small intestinal inguinal herniation

A

a) Strangulating pedunculated lipoma

(Strangulating > damaged RBCs > red / sanguinous fluid)

How well did you know this?
1
Not at all
2
3
4
5
Perfectly
25
Q

Which statement concerning small colon impactions is CORRECT?
a) If treated surgically there is a low risk of postoperative salmonella shedding
b) The aim of medical management is to soften the impaction, but diarrhoea may be a presenting sign
c) Can be localised to small colon based on the lack of a antimesenteric band
d) NSAID coverage and pain control are the most important aspects of treatment
e) Affected patients typically present with nasogastric reflux

A

b) The aim of medical management is to soften the impaction, but diarrhoea may be a presenting sign

How well did you know this?
1
Not at all
2
3
4
5
Perfectly
26
Q

Which of the following lists best describes the initial assessment of the cardiovascular system of a horse with colic in the field?
a. Palpate peripheral pulse; auscultate the heart; evaluate the mucous membranes and skin tent; assess the temperature of the extremities
b. Auscultate the heart and lungs; rectal temperature; evaluate the mucous membranes and skin tend; assess temperature pf the extremities
c. Palpate peripheral pulse; auscultate the heart; evaluate MMs; rectal temperature and auscultate the abdomen
d. Auscultate the heart and lungs; auscultate the abdomen; evaluate the MMS; check PCV, TS and lactate.

A

a. Palpate peripheral pulse; auscultate the heart; evaluate the mucous membranes and skin tent; assess the temperature of the extremities

How well did you know this?
1
Not at all
2
3
4
5
Perfectly
27
Q

A 4-year-old horse presents with a poor body condition (score 2/9) is dull and lethargic with watery diarrhoea. Blood analysis reveals albumin 16g?L (ref range 28-39g/L) globulin 44g/L (ref range 20-38g/L), urea 16mmol/L (ref range 3.2-8.1mmol/L), creatinine 0.22,,p;/L (ref range 0.08-0.16 mmol/L) and neutrophils 12.9g/L (ref range 2/8-8g/L).
Faecal analysis revealed no specific pathogens on culture and no helminth eggs.
Which of the following is the first and most important treatment for this horse?
a. IV fluid therapy
b. Antimicrobial treatment with trimethoprim sulphonamide
c. Withdraw food but offer fresh drink water with electrolytes
d. Supplement food with psyllium and Biosponge.

A

a. IV fluid therapy

How well did you know this?
1
Not at all
2
3
4
5
Perfectly
28
Q

Which of the following statements regarding air bronchograms is CORRECT?
a. An air bronchogram is a normal finding that has developed due to the increased contrast resolution that occurred when we shifted to digital imaging detectors
b. Air bronchograms are a sign of bronchial wall thickening and are indicative of bronchitis
c. Air bronchograms are the hallmark sigh of pleural effusion
d. Air bronchograms are air filled airways that become more visible in the lung periphery due to increased contrast when surrounded by the soft tissue opacity of alveolar pulmonary infiltrates.

A

d. Air bronchograms are air filled airways that become more visible in the lung periphery due to increased contrast when surrounded by the soft tissue opacity of alveolar pulmonary infiltrates.

How well did you know this?
1
Not at all
2
3
4
5
Perfectly
29
Q

Soft palate cautery is performed on the oro-pharyngeal surface of the soft palate rather than on the nasopharyngeal side Which of the following justifications is correct to support that statement?
a. It is less likely to damage the musculature of the soft palate
b. It is cheaper as it doesn’t require sophisticated equipment
c. it is safer as it doesn’t require general anaesthesia
d. it is creating much better sclerosis of the soft palate

A

a. It is less likely to damage the musculature of the soft palate

How well did you know this?
1
Not at all
2
3
4
5
Perfectly
30
Q

Image of endoscope of airway with white frothy mucous / liquid. Which of the following options describes the best sample to collect and the best diagnostic test (based on the most likely disease affecting this horse?)
a) Nasal swab for PCR
b) Needle aspirate of the retro-pharyngeal lymph node abscess for PCR
c) Nasopharyngeal lavage for bacterial culture
d) Needle aspirate of the retro-pharyngeal lymph node abscess for culture

A

a) Nasal swab for PCR

How well did you know this?
1
Not at all
2
3
4
5
Perfectly
31
Q

Which of the following upper respiratory tract obstructive disorders can be diagnosed on a resting endoscope examination?
a. Epiglottal retroversion
b. Lateral pharyngeal wall collapse
c. Dorsal displacement of the soft palate
d. Recurrent laryngeal neuropathy

A

b. Lateral pharyngeal wall collapse

How well did you know this?
1
Not at all
2
3
4
5
Perfectly
32
Q

Which of the following options include the most common clinical signs of Equine Gastric Ulcer Syndrome (EGUS)?
a. Colic, poor performance and poor haircoat
b. Diarrhoea, poor performance and weight loss
c. Partial anorexia, bruxism and colic
d. Partial anorexia, colic and changed behaviour

A

d. Partial anorexia, colic and changed behaviour

How well did you know this?
1
Not at all
2
3
4
5
Perfectly
33
Q

Diagram of inguinal hernia (looks like a testicle hanging down).
Most important muscle of an inguinal hernia?

A

Vaginal ring

How well did you know this?
1
Not at all
2
3
4
5
Perfectly
34
Q

The lung will naturally look more opaque in expiration as compared to inspiration.
a. True
b. False

A

True

How well did you know this?
1
Not at all
2
3
4
5
Perfectly
35
Q

Which of the following statements regarding “epidermal inclusions cysts (atheromas) is CORRECT?
a. Typically diagnosed using upper airway endoscopy
b. Can be treated using minimal invasive techniques in the field
c. Requires surgical intervention and is typically malignant
d. Typically affects older horses
e. Affects the dorsal conchal meatus

A

b. Can be treated using minimal invasive techniques in the field

How well did you know this?
1
Not at all
2
3
4
5
Perfectly
36
Q

Which of the following options best describe the ECG findings shown in the trace below? Looks like a normal rhythm (P:QRS ratio of 1:1) but some closer together & some further apart.
a. Supraventricular (atrial) premature complexes
b. Sinus arrhythmia
c. Second degree AV block
d. Ventricular premature complexes

A

b. Sinus arrhythmia

How well did you know this?
1
Not at all
2
3
4
5
Perfectly
37
Q

Which of the following are the most common pathogens associated with acute watery diarrhoea in horses?
a. Salmonella spp, Clostridium difficile and Rotavirus
b. Clostridium perfrinagens (Types A and C) Clostridium difficile and Corona virus
c. Clostridium difficile, Clostridium perfringens (A and C) and Clostridium septicum
d. Salmonella spp, clostridium difficile and clostridium perfingens (A and C)

A

d. Salmonella spp, clostridium difficile and clostridium perfingens (A and C)

How well did you know this?
1
Not at all
2
3
4
5
Perfectly
38
Q

A horse with colic has the following in-house lab results:
PCV 50%, TS 55g/L, serum lactate 5mmol/L.
Which of the following options is the MOST LIKELY interpretation of the results?
a) These laboratory results are mostly normal
b) Dehydration & splenic contraction
c) 10% dehydration & hypoproteinaemia
d) Splenic contraction & organ hypoperfusion

A

c) 10% dehydration & hypoproteinaemia

How well did you know this?
1
Not at all
2
3
4
5
Perfectly
39
Q

Which of the following options provides the cause of “high blowing” in the horse, the phase of respiration it occurs in and its clinical significance correctly?
a. Vibration of the nares during expiration, a normal finding
b. Vibration of the alar folds during inspiration, a normal finding
c. Vibration of the nares during inspiration, an abnormal finding causing airway obstruction
d. Vibration of the alar folds during expiration, an abnormal finding causing airway obstruction

A

d. Vibration of the alar folds during expiration, an abnormal finding causing airway obstruction

How well did you know this?
1
Not at all
2
3
4
5
Perfectly
40
Q

If a mass is seen in a location you have to consider what anatomic structure normally occupies that space & can become diseases. In regards to structures located within the thoracic mediastinum which of the following options is CORRECT?
- Cranial vena cava
- Heart
- None of these structures are in the mediastinum
- All of these structures are in the mediastinum
- Sternal lymph node
- Hilar lymph nodes
- Trachea
- Esophagus
- Caudal vena cava
- Thymus

A
  • All of these structures are in the mediastinum
How well did you know this?
1
Not at all
2
3
4
5
Perfectly
41
Q

Which of the following treatments is most effective for exercise induced pulmonary haemorrhage (EIPH)?
a. Oral administration of non-steroid anti-inflammatories to reduce airway inflammation
b. Application of nasal dilator strip during exercise
c. IV administration of a pro-coagulant such as aminocaproic acid to strenuous exercise
d. Intravenous administration of furosemide before strenuous exercise

A

d. Intravenous administration of furosemide before strenuous exercise

How well did you know this?
1
Not at all
2
3
4
5
Perfectly
42
Q

Pneumo-peritoneum (free air in the abdomen) has multiple findings on radiographs. Which of the following statements does NOT describe a typical finding of pneumo-peritoneum?
a. There is increased visibility of the intestinal walls
b. Severe segmental dilation of the small intestine develops (two populations of bowel).
c. Odd, angular gas bubbles are noted amongst the abdominal contents
d. A diaphragmatic stripe sign is present

A

b. Severe segmental dilation of the small intestine develops (two populations of bowel).

How well did you know this?
1
Not at all
2
3
4
5
Perfectly
43
Q

Which of the following statements is NOT correct with respect to development of ‘sharp points’ in the molar/premolar region of the equine mouth?
a. Are exacerbated by reduced grazing of coarse roughage
b. Occur on the lingual aspect of the maxillary arcade and the buccal aspect of mandibular arcade
c. Are the main reason regular routine dentistry is required
d. Can cause buccal ulceration
e. Occur on the buccal aspect of the maxillary arcade and the lingual aspect of the mandibular arcade

A

e. Occur on the buccal aspect of the maxillary arcade and the lingual aspect of the mandibular arcade

How well did you know this?
1
Not at all
2
3
4
5
Perfectly
44
Q

Which of the following options best describes the ECG findings shown in the image below from a horse identified with a cardiac arrhythmia at rest.
Several normal complexes followed by a complex that occurs early with wide & bizarre QRS.
a. Sinus arrhythmia
b. Ventricular premature complex
c. Second degree AV block
d. Supraventricular atrial premature complex

A

b. Ventricular premature complex

How well did you know this?
1
Not at all
2
3
4
5
Perfectly
45
Q

Myopathy and neuropathy are well-recognised potential complications of anaesthesia in horses. Which of the following is LEAST likely to prevent this complication from occurring?
a. Monitoring arterial blood pressure and maintain mean arterial blood pressure >70 mmHg
b. Use of appropriate padding and positioning of the limbs (e.g. down leg forward if in lateral recumbency).
c. Use of appropriate padding and positioning of the limbs (e.g. down leg forward if in lateral recumbency).
d. Removal of halters once the horse is recumbent

A

d. Removal of halters once the horse is recumbent

How well did you know this?
1
Not at all
2
3
4
5
Perfectly
46
Q

A horse presents with an extended head and neck carriage, intermittent nasal discharge, parotid region swelling and enlargement of the parotid and submandibular lymph nodes. Which of the following is the MOST likely diagnosis and most appropriate diagnostic testing respectively?
a. Guttural pouch mycosis; radiography of skull and cranial neck and ultrasonographic evaluation of the swollen regions
b. Guttural pouch empyema; radiography of skull and cranial neck and ultrasonographic evaluation of the swollen regions
c. Guttural pouch empyema; upper respiratory tract endoscopy and guttural pouch lavage for culture and sensitivity
d. Guttural pouch mycosis; upper respiratory tract endoscopy and guttural pouch lavage for culture and sensitivity testing

A

c. Guttural pouch empyema; upper respiratory tract endoscopy and guttural pouch lavage for culture and sensitivity

How well did you know this?
1
Not at all
2
3
4
5
Perfectly
47
Q

Which of the following is the most important in the after care of an emergency tracheotomy?
a. providing broad spectrum antimicrobials to prevent surgical site infection
b. daily cleaning and replacement of the tracheostomy tube and providing incisional care
c. providing broad spectrum antimicrobials to prevent lower airway infection
d. nebulizing the horse with bronchodilators to prevent further lower airway collapse

A

b. daily cleaning and replacement of the tracheostomy tube and providing incisional care

How well did you know this?
1
Not at all
2
3
4
5
Perfectly
48
Q

Which of the following findings on blood work would be present in a horse with a severe acute bacterial colitis?
a. leukopenia
b. hyperfibrinogenemia
c. Anaemia
d. Leucocytosis

A

d. Leucocytosis

(Increased WBCs due to bacterial infection)

How well did you know this?
1
Not at all
2
3
4
5
Perfectly
49
Q

A grade 3 holo-diastolic, decrescendo murmur with the point of maximal intensity over the left heart base is auscultated in a 16-year old eventer. Which of the following options is the most likely origin of this murmur?
a. tricuspid regurgitation
b. aortic regurgitation
c. physiological pulmonary ejection murmur
d. mitral regurgitation

A

b. aortic regurgitation

How well did you know this?
1
Not at all
2
3
4
5
Perfectly
50
Q

What volume and rate of isotonic crystalloid fluids would be most appropriate for initial resuscitation of a 50kg foal with signs of hypovolaemic shock?
a. 2-3 litres IV, given as 1 litre boluses, with frequent reassessment of hydration status.
b. 3 litres of Hetastarch© given over 20-40mins
c. 2.5 litres given over 6h, followed by 2.5 v
d. 0.5-1.5 litres orally per nasogastric tube given over 6 hrs

A

?

How well did you know this?
1
Not at all
2
3
4
5
Perfectly
51
Q

Fill in the blanks… Radiographic signs of broncho-pneumonia are usually a(an) (blank) pattern distributed (blank). Which of the following answer combinations is most CORRECT?
a. Interstitial, dorsally
b. Alveolar, ventrally
c. Alveolar, diffusely
d. Alveolar, dorsally
e. Bronchial, diffusely
f. Bronchial, cranio-dorsally
g. Interstitial, diffusely
h. Bronchial, cranio-ventrally

A

c. Alveolar, diffusely

How well did you know this?
1
Not at all
2
3
4
5
Perfectly
52
Q

Thoracic radiographs performed after trauma can be confusing because multiple common lesions can occur simultaneously. I taught you to consider a list of five traumatically induced lesions that occur with thoracic blunt trauma to assist your interpretation. Which of the following lesions is NOT included in this list because it does not commonly occur with trauma?
a. Rib fracture
b. Pericardial effusion
c. Diaphragmatic hernia
d. All of the above
e. Pulmonary contusions
f. Pneumothorax
g. None of the above
h. Pleural effusion

A

b. Pericardial effusion

How well did you know this?
1
Not at all
2
3
4
5
Perfectly
53
Q

During total intravenous anaesthesia, assessing anaesthesia depth can be difficult in horses. Which of the following variables provide you with the LEAST valuable information regarding depth of anaesthesia?
a. Muscle tone
b. Arterial blood pressure
c. Heart rate
d. Presence of nystagmus

A

a. Muscle tone

How well did you know this?
1
Not at all
2
3
4
5
Perfectly
54
Q

In a horse with acute, severe colic (photo on left), 200mg of xylazine is administered to allow a brief clinical examination (mucous membranes are shown on the right) but the horse remains uncomfortable. What is the first thing you should do next?

A

Has toxic line - hyperaemic around gums > endotoxaemic

Early aggressive treatment - IV fluids, AB treatment (broad spectrum approach). Antitoxin (if toxic event). Surgery if stable.

How well did you know this?
1
Not at all
2
3
4
5
Perfectly
55
Q

Which combination of clinical and laboratory findings would be MOST supportive of the need for fluid therapy, in a horse you suspect is hypovolaemic?
a. Increased Heart Rate, Decreased PCV, Increased respiratory rate, Increased creatinine
b. Increased Heart Rate, Increased PCV, Decreased anion gap, Decreased capillary refill time
c. Increased Heart Rate, Increased PCV, Increased blood lactate, Increased capillary refill time
d. Normal heart rate, Increased PCV, normal blood lactate, Increased rectal temperature

A

c. Increased Heart Rate, Increased PCV, Increased blood lactate, Increased capillary refill time

How well did you know this?
1
Not at all
2
3
4
5
Perfectly
56
Q

Which of the following are the morphological and possible physiological characteristics of a premature/dysmature foal?
a. Morphological: overgrown foal, floppy ears, domed forehead and short and fine hair coat. Physiological: immature lungs, normal GI tract, poor thermoregulation, wear or normal suckle reflex, entropion.
b. Morphological: thin, floppy ears, cleft palate and short fine hair coat. Physiological: immature lungs, immature Gi tract, poor thermoregulation, weak or normal suckle reflex, normal eyes.
c. Morphological: thin, floppy ears, domed head and short and fine hair coat. Physiological: immature lungs, immature GI tract, poor thermoregulation, weak or normal suckle reflex, entropion.
d. Morphological: thin, floppy ears, cleft palate and short and fine hair coat. Physiological: immature lungs, immature Gi tract, poor thermoregulation, weak or normal suckle reflex, normal eyes.

A

c. Morphological: thin, floppy ears, domed head and short and fine hair coat. Physiological: immature lungs, immature GI tract, poor thermoregulation, weak or normal suckle reflex, entropion.

How well did you know this?
1
Not at all
2
3
4
5
Perfectly
57
Q

Which of the following cell types are most likely to be elevated in the respiratory secretions of a horse with moderate equine asthma?
a. eosinophils
b. macrophages
c. lymphocytes
d. neutrophils

A

d. neutrophils

How well did you know this?
1
Not at all
2
3
4
5
Perfectly
58
Q

Which of the following statements regarding Biosponge® (di-trioctahedral smectite) is CORRECT?
a. It binds to bacterial toxins within the gastrointestinal lumen and reduces bacterial translocation into circulation.
b. Several cases of Di-trioctahedral smectite associated with toxicosis in horses have been reported.
c. No studies support Di-trioctahedral smectite use in equine medicine
d. It has a prokinetic mechanism of action thereby increasing gastrointestinal transit time and reducing the frequency diarrhoea.

A

a. It binds to bacterial toxins within the gastrointestinal lumen and reduces bacterial translocation into circulation.

How well did you know this?
1
Not at all
2
3
4
5
Perfectly
59
Q

Diagram of anus with rectal tear in submucosa / mucosa. What is the most appropriate treatment?
a) Antimicrobials, laxatives, daily inspection and evacuation of rectum
b) Immediate euthanasia due to grave prognosis
c) Antimicrobials, NSAIDs, rectal enema and referral
d) Antimicrobials, NSAIDs, suture of tear and rectal packing
e) Antimicrobials, laxatives, rectal liner, daily inspection and evacuation of rectum

A

c) Antimicrobials, NSAIDs, rectal enema and referral

How well did you know this?
1
Not at all
2
3
4
5
Perfectly
60
Q

Photo of very obvious V-shaped epiglottis. Which of the following is the most likely diagnosis?
a) Epiglottal entrapment
b) Subepiglottal cyst

A

a) Epiglottal entrapment

How well did you know this?
1
Not at all
2
3
4
5
Perfectly
61
Q

Fill in the blanks: A 48 hour old colt is dull and not nursing well. Haematology reveal a moderate leukopenia with a left shift neutropenia, moderate toxic change and an IgG concentration of 5g/L.
a) What is the most likely differential diagnosis?
b) What diagnostic test would help you confirm your suspicion?

A

a) Failure of passive transfer
b) Antibody Test (IgG)

How well did you know this?
1
Not at all
2
3
4
5
Perfectly
62
Q

Which of the following lists best describes the initial assessment of the cardiovascular system of a horse with colic, in the field?
a. auscultates the heart and lungs; rectal temperature; elevate the mucous membranes and skin tent; assess the temperature of the extremities.
b. Palpate peripheral pulse; auscultate the heart; evaluate the mucous membranes and skin tent; assess the temperature of the extremities
c. Auscultate the heart and lungs; auscultate the abdomen; evaluate the mucous membranes; check PCV, TS and lactate.
d. Palpate peripheral pulse; auscultate the heart; evaluate mucous membranes; rectal temperature and auscultate the abdomen.

A

a. auscultates the heart and lungs; rectal temperature; elevate the mucous membranes and skin tent; assess the temperature of the extremities.

How well did you know this?
1
Not at all
2
3
4
5
Perfectly
63
Q

A neonatal foal hasn’t been observed to urinate since foaling (approximately 24 hours ago) and a ruptured bladder is suspected. Which of the following diagnostic findings confirms the diagnosis of a uroperitoneum?
a. Transabdominal ultrasonography and with evidence of an increase in free peritoneal fluid
b. Urine analysis and culture
c. Systemic creatinine two times higher than peritoneal fluid creatinine
d. Peritoneal fluid creatinine two times higher than systemic creatinine.

A

d. Peritoneal fluid creatinine two times higher than systemic creatinine.

How well did you know this?
1
Not at all
2
3
4
5
Perfectly
64
Q

List two differential diagnoses for the following trans-cutaneous ultrasonographic finding(s) of a mare with acute, moderate to severe signs of colic with injected and tacky mucous membranes and a CRT of 3-4 sec. (0.5 marks each)

A
  • Intussusception
  • Peritonitis
How well did you know this?
1
Not at all
2
3
4
5
Perfectly
65
Q

Which combination of clinical and laboratory findings from a horse suspected to be hypovolemic would be MOST supportive for the need for fluid therapy?

A

Increased heart rate, increased PCV, increased blood lactate

How well did you know this?
1
Not at all
2
3
4
5
Perfectly
66
Q

A 500kg thoroughbred mare has had profuse water diarrhoea for 2 days and is now estimated to be 6% dehydrated. What volume of fluid is needed to correct this fluid deficit?
a) 10L
b) 100L
c) 15L
3) 30L

A

?

How well did you know this?
1
Not at all
2
3
4
5
Perfectly
67
Q

How can hypovolemia be best distinguished from dehydration when performing a clinical examination?
a) The horse is mildly depressed, skin turgor 2 sec and heart rate 52 beats per minute
b) The horse is depressed, tacky mucous membranes and skin turgor 1 sec
c) The horse is depressed, mucous membranes are tacky and CRT 2 sec
d) The horse is depressed, CRT 2-3s and cold extremities

A

d) The horse is depressed, CRT 2-3s and cold extremities

(Horses with hypovolemia show signs of loss of circulating IV fluid including depression, cold extremities, increased capillary refill time)

How well did you know this?
1
Not at all
2
3
4
5
Perfectly
68
Q

You have decided to administer IV fluids to a horse for management of dehydration. Which ONE of the following findings would indicate that your fluid replacement plan had been successful?
a) The horse’s rectal temperature returns to normal
b) The horse starts drinking from its water bucket
c) The horse’s appetite returns
d) The horse starts urinating

A

d) The horse starts urinating

(Urination is a good indicator of adequate circulating fluid volume in response to fluid therapy. Monitoring the specific gravity of the urine can also indicate adequate hydration (USG will normalise, having been increased in the dehydrated state)

How well did you know this?
1
Not at all
2
3
4
5
Perfectly
69
Q

Which is the most appropriate type and route of fluid administration to an adult horse with profuse, watery diarrhoea and signs of hypovolemia?
a) Synthetic colloid given orally
b) Isotonic bicarbonate fluid given IV
c) Isotonic crystalloid fluid given IV
d) Water with 5% dextrose given orally

A

c) Isotonic crystalloid fluid given IV

How well did you know this?
1
Not at all
2
3
4
5
Perfectly
70
Q

What is the most appropriate fluid therapy regimen for a cardiovascularly stable 500kg horse with an uncomplicated impaction of the left ventral colon?
a) Encourage access to free water containing salt solution
b) Repeat fluid boluses of 8-10 litres isotonic fluid given via nasogastric tube
c) Continuous rate enteric fluid therapy with hypotonic fluids at a rate of 5L/hr via an indwelling nasogastric tube
d) Repeat administration of 20 litre fluid bolus of IV crystalloid solution

A

b) Repeat fluid boluses of 8-10 litres isotonic fluid given via nasogastric tube

(Aggressive enteric fluid therapy with frequent doses of isotonic fluids via NGT has been shown to be the most effective (& economic) method of clearing colonic impactions in horses)

How well did you know this?
1
Not at all
2
3
4
5
Perfectly
71
Q

What is the most appropriate fluid therapy / regime for a 500kg horse with hypovolemia (8-10% dehydration)?
a) IV administration of 5 litres Hetastarch (synthetic colloid solution) and crystalloid fluids at maintenance rates
b) Oral fluids by nasogastric tube 8-10L/hour
c) IV fluids at 4-6L/hr
d) Drinking water with electrolytes (salts 4.9g/L)

A

c) IV fluids at 4-6L/hr

(This degree of hypovolemia reflects a marked loss of circulating fluid volume and requires at least 4-6L per hour (4-6 x maintenance fluid rates) during the first 12 hours to restore the fluid deficit. Alternatively, a bolus of 80ml/kg (40L) could be given followed by a fluid rate of 2-3L per hour)

How well did you know this?
1
Not at all
2
3
4
5
Perfectly
72
Q

What volume and rate of isotonic crystalloid fluid would be most appropriate for initial fluid replacement for a 50kg foal with signs of hypovolemic shock?
a) 500-1500ml orally per nasogastric tube given once
b) 2-5L IV given as increments of 1L boluses with frequent reassessment of hydration status
c) 5L IV given over 20-40 mins
d) 2.5L given over 6h followed by 2.5L over the next 12hr

A

b) 2-5L IV given as increments of 1L boluses with frequent reassessment of hydration status??

How well did you know this?
1
Not at all
2
3
4
5
Perfectly
73
Q

Calculate the approximate maintenance fluid requirement rate (over 12 hours) for a 500kg horse that has ongoing losses of 6L every 2 hours due to gastric reflux.
a) 24L (2L/hr)
b) 36L (3L/hr)
c) 12-18L (1-1.5L/hr)
d) 49-54L (4-5L/hr)

A

?

How well did you know this?
1
Not at all
2
3
4
5
Perfectly
74
Q

Which is the most appropriate clinical indication for the administration of hypertonic (7.2%) saline?
a) An adult horse showing signs of hypovolemic shock
b) A mildly dehydrated horse
c) A neonatal foal showing signs of hypovolemic shock
d) An endurance horse with signs of significant dehydration

A

?

How well did you know this?
1
Not at all
2
3
4
5
Perfectly
75
Q

Which of the following is most consistent with biopsy report for a horse suffering clinical signs of liver disease from Senecio spp. ingestion?
a) Biliary hyperplasia, periportal fibrosis, megalocytosis
b) Centrilobar necrosis
c) Periportal hepatocellular necrosis and bridging fibrosis
d) Bile duct stasis, suppurative cholangitis

A

a) Biliary hyperplasia, periportal fibrosis, megalocytosis

How well did you know this?
1
Not at all
2
3
4
5
Perfectly
76
Q

Which of the following biochemical abnormalities would you expect from a horse suffering choledocholelithiasis?
a) Elevated GGT, elevated total bilirubin
b) Elevated GGT, elevated SDH
c) Elevated AST, elevated SDH
d) Elevated AST, normal GGT

A

a) Elevated GGT, elevated total bilirubin

How well did you know this?
1
Not at all
2
3
4
5
Perfectly
77
Q

Which of the following is correct for a pony suffering hepatic lipidosis?
a) Insulin resistance promotes peripheral lipolysis which potentiates hyperlipaemia
b) Ketones are typically elevated in urine
c) The liver is typically small and cirrhotic at necropsy
d) The failure of hormone sensitive lipase to clear VLDL from circulation leads to lipaemia

A

a) Insulin resistance promotes peripheral lipolysis which potentiates hyperlipaemia

How well did you know this?
1
Not at all
2
3
4
5
Perfectly
78
Q

What is the most common cause of photodermatitis in Australia (direct or indirect)?
a) Salvation Jane plant causing liver disease
b) Blue Heliotrope plant causing liver disease
c) Direct photodermatitis unrelated to liver disease
d) Oleander plant causing liver disease

A

a) Salvation Jane plant causing liver disease

How well did you know this?
1
Not at all
2
3
4
5
Perfectly
79
Q

What are the classical histopathology findings of a liver biopsy of horses with pyrrolizidine alkaloid toxicity?
a) Megalocytosis, periportal fibrosis and bile duct hyperplasoa
b) Megalocytosis, bile duct fibrosis and hyperplasia
c) Periportal fibrosis, bile stone and bile duct hyperplasia
d) Lymphocytes and neutrophils, periportal fibrosis and bile duct hyperplasia

A

a) Megalocytosis, periportal fibrosis and bile duct hyperplasoa

How well did you know this?
1
Not at all
2
3
4
5
Perfectly
80
Q

Describe your initial diagnostic plan for a horse with suspected liver disease (no neurlogic signs noted).
a) Specific and non-specific liver enzyme activities, liver ultrasound and liver biopsy
b) Specific liver enzyme activities, liver palpation and liver biopsy
c) Specific and non-specific liver enzyme activities, liver ultrasound and blood anaemia concentration
d) Liver function tests, liver ultrasound and liver biopsy

A

a) Specific and non-specific liver enzyme activities, liver ultrasound and liver biopsy

How well did you know this?
1
Not at all
2
3
4
5
Perfectly
81
Q

Which of the following tests do you consider important during your diagnostic testing in a horse suspected of liver disease:
a) AST, GGT, bile acids and bilirubin
b) AST, GLDH, bile acids and ammonia
c) AST, CK, bilirubin and ammonia
d) CK, GGT, bile acids and bilirubin

A

a) AST, GGT, bile acids and bilirubin

How well did you know this?
1
Not at all
2
3
4
5
Perfectly
82
Q

Which IV fluid would not be appropriate to administer to a hypovolaemic horse?
a) Plasma
b) Dextrose 50%
c) 0.9% saline
d) Hypertonic saline

A

b) Dextrose 50%

How well did you know this?
1
Not at all
2
3
4
5
Perfectly
83
Q

A 14 year old QH is presented with liver disease, liver biopsy reveals biliary stasis and neutrophilic inflammation. What is your treatment?
a) Antimicrobials and supportive therapy
b) Anti-inflammatory and high protein diet
c) Surgery and supportive therapy
d) Antimicrobials and sedative therapy

A

a) Antimicrobials and supportive therapy

How well did you know this?
1
Not at all
2
3
4
5
Perfectly
84
Q

Which statement regarding acute colitis is false?
a) Prostaglandins such as PGI2 & PGE2 may be cytoprotective to gastrointestinal mucosa
b) Horses with severe colitis may have profound hypoproteinaemia
c) Metronidazole use is indicated in cases of C. difficile infection and neutropenia
d) Flunixin meglumine must be administered at 1.1mg/kg BW IV q12h to decrease production of tumour necrosis factor and other inflammatory cytokines in the GI mucosa

A

d) Flunixin meglumine must be administered at 1.1mg/kg BW IV q12h to decrease production of tumour necrosis factor and other inflammatory cytokines in the GI mucosa

How well did you know this?
1
Not at all
2
3
4
5
Perfectly
85
Q

Which commonly used therapy lacks experimental support as a beneficial treatment of acute colitis caused by salmonellosis?
a) Lactobaccilus pentosus
b) Penicillin
c) Biosponge
d) Probiotic pastes

A

b) Penicillin

How well did you know this?
1
Not at all
2
3
4
5
Perfectly
86
Q

Which statement regarding IV fluid therapy for acute colitis is NOT CORRECT?
a) Potassium should be supplemented carefully
b) Fluids should not be administered until clinicopathologic results are available
c) Potassium supplementation is often required due to reduced feed intake and increased GI loss
d) Ongoing fluid losses must be considered in addition to maintenance requirements and replacement of los fluid (diarrhoea) when determining an appropriate fluid plan

A

b) Fluids should not be administered until clinicopathologic results are available

How well did you know this?
1
Not at all
2
3
4
5
Perfectly
87
Q

Which statement regarding acute colitis is NOT CORRECT?
a) If complications do not occur and diarrhoea resolves, the horse should have no longterm effects
b) Antimicrobial therapy should not be routinely administered in colitis cases
c) Laminitis is an uncommon sequela of acute colitis
d) Many components of treatment are the same regardless of aetiology

A

c) Laminitis is an uncommon sequela of acute colitis

How well did you know this?
1
Not at all
2
3
4
5
Perfectly
88
Q

Which statement regarding treatment of acute colitis is NOT CORRECT?
a) Antimicrobial therapy may prolong bacterial shedding in salmonella cases
b) There is evidence to support the use of transfaunation per nasogastric tube
c) Treatment with NSAIDs should be routine because they limit the chance of developing intestinal ulceration
d) Oxytetracycline is an appropriate therapy for salmonellosis cases

A

c) Treatment with NSAIDs should be routine because they limit the chance of developing intestinal ulceration

How well did you know this?
1
Not at all
2
3
4
5
Perfectly
89
Q

Horses with a ventricular septal defect have a:
a) Systolic murmur on the left side, PMI in the apex area
b) Diastolic murmur on the right side, PMI over the heart base
c) Diastolic murmur on the left side, PMI over the heart base
d) Systolic murmur on the right side, PMI in the apex area

A

d) Systolic murmur on the right side, PMI in the apex area

How well did you know this?
1
Not at all
2
3
4
5
Perfectly
90
Q

Examine the following ECG (normal waves & then a p wave without a QRS complex after it). What is your diagnosis?

A
  • 2nd degree AV block
How well did you know this?
1
Not at all
2
3
4
5
Perfectly
91
Q

Which of the following is the most common cause of a systolic murmur on the right side of the thorax in the horse?
a) Ventricular septal defect
b) Aortic regurgitation
c) Tricuspid regurgitation
d) Mitral regurgitation

A

c) Tricuspid regurgitation

How well did you know this?
1
Not at all
2
3
4
5
Perfectly
92
Q

What is the most likely cause of a holosystolic murmur auscultated over the left apex and radiating dorsally?
a) Pulmonic valve insufficiency
b) Aortic valve insufficiency
c) Mitral valve regurgitation
d) Flow murmur

A

c) Mitral valve regurgitation

How well did you know this?
1
Not at all
2
3
4
5
Perfectly
93
Q

Which of the following is most important in the ongoing maintenance of equine asthma?
a) Reduction of respirable dust levels
b) Reduced exercise intensity / retirement from strenuous exercise
c) Daily treatment with inhaled corticosteroids
d) Daily nebulisation with a bronchodilator before exercise

A

a) Reduction of respirable dust levels

How well did you know this?
1
Not at all
2
3
4
5
Perfectly
94
Q

Which diagnostic tool is most useful to confirm a definitive diagnosis of equine asthma?
a) Bronchoalveolar lavage
b) Tracheal wash
c) Endoscopy
d) Nasopharyngeal swab

A

a) Bronchoalveolar lavage

How well did you know this?
1
Not at all
2
3
4
5
Perfectly
95
Q

Review the attached endoscopic image and grade the degree of mucous observed.

A

Grade 3

How well did you know this?
1
Not at all
2
3
4
5
Perfectly
96
Q

Review the following cytology slide from a broncho alveolar lavage and identify the cells labelled A, B and C.

A

A = macrophage, B = neutrophil, C = eosinophil

How well did you know this?
1
Not at all
2
3
4
5
Perfectly
97
Q

Review the tracheal wash cytology slide and identify the pink material in the background.

A

Mucous

How well did you know this?
1
Not at all
2
3
4
5
Perfectly
98
Q

In a horse with recurrent laryngeal neuropathy which muscle is implicated in the failure to abduct the arytenoid cartilage?

A

Cricoarytenoid dorsalis

How well did you know this?
1
Not at all
2
3
4
5
Perfectly
99
Q

Which of the following forms of dynamic airway collapse is most likely to be associated with abnormal respiratory sounds during expiration?
a) Dorsal displacement of the soft palate
b) Vocal fold collapse
c) Pharyngeal wall collapse
d) Axial deviation of the aryepiglottal folds

A

a) Dorsal displacement of the soft palate

How well did you know this?
1
Not at all
2
3
4
5
Perfectly
100
Q

Which of the following conditions is a common cause of chronic cough in the adult horse?
a) Lungworm
b) Tracheal collapse
c) Oesophageal obstruction
d) Equine asthma

A

d) Equine asthma

101
Q

Guaifenesin is a central acting muscle relaxant used in Triple Drip combinations for equine field anaesthesia. From the following list choose 2 other drugs that guaifenesin is commonly combined with to make up Triple Drip.

A

Xylazine & Ketamine

102
Q

Which factor is NOT directly associated with increased risk for postoperative myopathy in the horse?
a) Acepromazine
b) Intraoperative hypotension
c) Position on the table
d) Anaesthetic time

A

a) Acepromazine

103
Q

The owner of a horse undergoing colic surgery (with an apparently poor prognosis) decides to euthanise the horse while still under general anaesthesia. What would be best method of euthanasia?
a) IV barbiturate
b) Captive bolt
c) IV potassium chloride
d) Intrathecal lignocaine

A

a) IV barbiturate

104
Q

What would be the best parameter to confirm death in a horse that has been euthanised?
a) Rigor mortis
b) Absence of cardiac activity
c) Apnea
d) No palpable arterial pulse

A

b) Absence of cardiac activity

105
Q

From the list below select the best description of when your horse is ready to have induction drugs administered.
a) Heart rate below 30bpm, RR below 10bpm
b) Eyelids closed and ears back
c) Knuckling in forelimbs, hind limbs crossed
d) Head hanging down, wide based stance

A

d) Head hanging down, wide based stance

106
Q

What is the electrical cardiac abnormality depicted in this ECG and which drug can cause this (p wave followed by no QRS complex).

A

2nd degree AV block - detomidine

107
Q

Which rules should be included in the isolation protocol once strangles is diagnosed at a stable yard?

A

Isolation of horses up to 3 weeks after resolution of clinical signs of strangles disease and negative culture of naso-pharyngeal swabs

108
Q

Which of the following upper airway obstructive conditions is most likely to be associated with expiratory “gurgling” sounds?
a) Dorsal pharyngeal wall collapse
b) Dorsal displacement of the soft palate
c) Tracheal collapse
d) Recurrent laryngeal neuropathy

A

b) Dorsal displacement of the soft palate

(DDSP is one of the few upper airway obstructions that results in expiratory sounds.)

109
Q

Which of the following is necessary for a DEFINITIVE diagnosis of Rhodococcus equi in foals?
a) Ultrasound evidence of bronchopneumonia along with clinical signs of lower resp tract disease
b) Bacterial culture or PCR of VapA gene from trans tracheal wash or tracheobronchial aspirate, along with clinical signs of lower respiratory tract disease
c) Fungal culture or PCR of RhodE gene from trans tracheal wash or tracheobronchial aspirate, along with clinical signs of lower resp tract disease
d) Radiographic evidence of bronchopneumonia along with clinical signs of lower resp tract disease
e) Cytological evidence of septic airway inflammation, along with clinical signs of lower resp tract disease

A

b) Bacterial culture or PCR of VapA gene from trans tracheal wash or tracheobronchial aspirate, along with clinical signs of lower respiratory tract disease

110
Q

What is the aetiology of strangles and how does the disease spread?
a) Streptococcus equi subspecies zooepidemicus which is transmitted only by direct contact
b) Streptococcus equi subspecies zooepidemicus which is transmitted either by direct or indirect contact
c) Streptococcus equi subspecies equi, which is transmitted either by direct or indirect contact
d) Streptococcus equi subspecies equi which is transmitted only by direct contact

A

c) Streptococcus equi subspecies equi, which is transmitted either by direct or indirect contact

111
Q

Image of upper airway endoscopic examination. Whole area including epiglottis is bright red / haemorrhagic discharge.
Most likely differential?

A
  • Guttural pouch mycosis
112
Q

Radiographic image of horses head with ltos of white whispy lines. Name the most apparent radiographic finding or diagnosis in the xray below.
a) Mass at the base of the ethmoid turbinates
b) Tympany of the guttural pouch
c) Fluid lines in sinuses
d) Arytenoid chondritis
e) Chondroids within the guttural pouch

A

c) Fluid lines in sinuses

113
Q

Which of the following statements about emergency tracheostomy in horses is false?
a) It is easier to transversely transect the ligament between two adjacent tracheal rings
b) Once the primary condition is controlled the mucosa of the trachea is sutured to promote healing
c) The procedure doesn’t always necessitate local anaesthesia and sedation
d) It is important to resect no more than half of two adjacent tracheal rings to avoid tracheal collapse

A

b) Once the primary condition is controlled the mucosa of the trachea is sutured to promote healing

(emergency tracheostomy is usually left to heal by second intention)

114
Q

Which of the following is most important to prevent recurrence of clinical signs in a horse with moderate equine asthma?
a) Inhaled corticosteroids to reduce the inflammatory response
b) Regular nebulisation with bronchodilators
c) Reduction of respirable dust levels, especially in feed
d) A course of antimicrobials (per os) to reduce overgrowth of commensals

A

c) Reduction of respirable dust levels, especially in feed

115
Q

Examine the following endoscopic image taken during exercise (epiglottis & arytenoids in hourglass shape). What form of dynamic airway collapse is shown?

A
  • Axial deviation of the aryepiglottal folds
116
Q

Which of the following statements is correct with regards to atrial fibrillation in horses?

A
  • Quinidine Sulphate is a class I antiarrhythmic medication used for pharmacological cardioversion of atrial fibrillation in horses
117
Q

A 7 year old warmblood gelding used for advanced level dressage is presented with a very loud respiratory noise without exercise intolerance. Grade III laryngeal dysfunction is diagnosed during resting endoscopic examination. What treatment is recommended?
a) A standing bilateral trans-endoscopic ventriculo-cordectomy using a diode laser
b) A prosthetic laryngopalsty (“tie back”) performed on the horse under standing sedation
c) A prosthetic laryngoplasty (“tie back”) performed on the horse under GA
d) A 6 week course of oral dexamethasone with dust management of the environment

A

?

118
Q

Image of horse with white mucousy discharge coming out of one nostril. Which of the following diagnoses is LEAST likely when presented with the clinical finding depicted?
a) Primary sinusitis
b) Apical tooth root infection
c) Epidermal inclusion cyst / atheroma
d) Guttural pouch empyema
e) Progressive ethmoid haematoma

A

?

119
Q

What is the most likely diagnosis for a horse that is presented with bilateral purulent nasal discharge and fever without a cough?
a) Strangles disease
b) Pleuropneumonia
c) Equine asthma
d) Bronchopneumonia

A

? (not bronchopneumonia)

120
Q

Laryngeal ultrasonography is a useful diagnostic aid in which of the following conditions?
a) Arytenoid chondritis
b) Pharyngeal wall collapse
c) Epiglottal entrapment
d) Intermittent dorsal displacement of the soft palate

A

? (not dorsal displacement of soft palate)

121
Q

Which of the following is the most common cause of systolic murmurs on the right side of the thorax in the horse?
a) Mitral regurgitation
b) Ventricular septal defect
c) Aortic regurgitation
d) Tricuspid regurgitation

A

d) Tricuspid regurgitation

122
Q

A 5 year old thoroughbred gelding presents to you with unilateral chronic mucopurulent discharge from the right nostril. What is the predominating finding / diagnosis in the transverse CT image of this animal below?
a) Epidermal inclusion cyst of the right middle meatus
b) Tympany of the right caudal maxillary sinus
c) Sinusitis of the right ventral conchal and maxillary sinuses

A

c) Sinusitis of the right ventral conchal and maxillary sinuses

123
Q

Which of the following is the most likely source of bleeding in a horse with exercise induced pulmonary haemorrhage?
a) Bronchial vasculature; cranioventral lung lobes
b) Bronchial vasculature; caudodorsal lung lobes
c) Primary vasculature; cranioventral lung lobes
d) Pulmonary vasculature; caudodorsal lung lobes

A

d) Pulmonary vasculature; caudodorsal lung lobes

124
Q

Horses must be adequately sedated prior to induction of field anaesthesia. Which statement best describes the reason for this?
a) Profound sedation provides a stable cardiovascular status, minimising the anaesthesia related risk during the procedure
b) Adequate analgesia accompanies profound sedation, which is beneficial for painful procedures
c) Amnesia accompanies profound sedation and improves the quality of induction
d) Adequate sedation provides a margin of safety during the relatively uncontrolled induction

A

? (not B)

125
Q

In cases of pneumonia or pleuropneumonia positive for Bacteroides species (anaerobic bateria) on culture, which of the following antimicrobials should be included in the therapy?
a) Gentamicin
b) Penicillin
c) Enrofloxacin
d) Metronidazole
e) Amikacin

A

? (not penicillin)

126
Q

Horse with fleshy chunk below its eye. Choose the correct statement.
a) An open fracture and laceration into the rostral maxillary sinus
b) The prognosis for a full recovery is poor
c) An open fracture and laceration into the ethmoid turbinates
d) A full thickness laceration into the oral cavity
e) Drainage and second intention healing are indicated

A

e) Drainage and second intention healing are indicated

127
Q

What is the main cause of hypoxaemia in horses undergoing colic surgery?
a) Anaemia
b) Lung oedema
c) Hypoventilation
d) Lung atelectasis

A

c) Hypoventilation

128
Q

Which of the following is the most likely life threatening side effect in horses medicated with phenylbutazone?
a) Proximal enteritis
b) Right dorsal colitis
c) Gastric ulceration
d) Anaphylaxis

A

c) Gastric ulceration

129
Q

Non-steroidal anti-inflammatory (NSAIDs) exert their anti-inflammatory effect through inhibition of:
a) The cyclo-oxygease pathway
b) Neither the cyclo-oxygenase and lipo-oxygenase pathway
c) Both the cyclo-oxygenase and lipo-oxygenase pathway
d) The lipo-oxygenase pathway

A

a) The cyclo-oxygease pathway

130
Q

Eruption of permanent incisor 203 has occured by what age in the horse?

A

5 years

131
Q

A 500kg thoroughbred mare has had profuse watery diarrhoea for two days and is now estimated to be 6% dehydrated. What volume of fluid is needed to correct this fluid deficit?

A

30L

132
Q

What clinical feature is the most sensitive indicator of adequate fluid replacement in profoundly dehydrated horses?
a) 50% reduction in serum lactate concentration
b) Increasing arterial blood pressure
c) Normalisation of packed cell volume (PCV) and total serum protein (TSP) concentration
d) Resumption of normal urine output

A

d) Resumption of normal urine output

133
Q

Calculate the approximate maintenance fluid requirement rate (over 12 hours) for a 500kg horse that has ongoing losses of 6L every 2 hours due to gastric reflux.

A

36L (3L/hr)

134
Q

Inclement weather (low temperatures, wind and rain) may lead to weight loss because:
a) It increases the competition for food among herd mates
b) Horses are unable to digest and absorb nutrients as efficiently
c) It increases the digestible energy reqruirements
d) It induces partial anorexia

A

c) It increases the digestible energy reqruirements

135
Q

A horse with protrusion of the third eyelid, a “saw horse stance” elevated tail head & demonstrating dysphagia is most likely to be suffering from?

A

Tetanus

136
Q

Dysphagia and muscle fasciculations in a horse without fever is most likely cause by:
a) Hyperkalaemic periodic paralysis (HYPP)
b) Hendra virus infection
c) Exertional rhabdomyolysis
d) Kunjin encephalitis virus infection

A

d) Kunjin encephalitis virus infection

137
Q

A horse showing hind limb ataxia, bladder paresis, perineal hypalgesia but normal mentation and no muscle tremors is most likely to have been infected by which virus?
a) Kunjin virus
b) Equine herpes virus 1
c) Rabies virus
d) Ross rive virus

A

b) Equine herpes virus 1

138
Q

In relation to the detection of a patent adult tapeworm infestation in an individual hrose the McMasters faecal egg count can be considered as having:

A
  • High sensitivity and low specificity
139
Q

Which of the following anthelmintic drugs is effective against encysted larval stages of the Cyathastome nematodes?
a) Ivermectin
b) Praziquantel
c) Oxfendazole
d) Moxidectin

A

d) Moxidectin

140
Q

What is the most important technical aspect of an episioplasty on a maiden-mare to ensure a long breeding career?
a) Perform a wide breeding stitch
b) Excise only a thin muscle layer
c) Use an absorbable suture material
d) Excise only a thin muco-cutaneous layer

A

d) Excise only a thin muco-cutaneous layer

141
Q

Which two disease categories account for approximately 50% of cases of weight loss in adult horses?
a) Parasites and dental disorders
b) Hepatopathy and neoplasia
c) Digestive disorders and renal disease
d) Abdominal infection and PPID / Cushing’s disease

A

a) Parasites and dental disorders

142
Q

Which would be the most economical first approach to a weight loss case without any localising signs of disease?
a) Rectal exam, abdominal ultrasonography, abdominocentesis, complete blood cell count and biochemistry analysis
b) Dental examination, rebreathing examination, faecal egg count, complete blood cell count and biochemistry analysis
c) Dental examination, rectal exam, faecal egg count, abdominal ultrasonography and abdominocentesis
d) Abdominal radiographs, rectal exam, rebreathing examination, faecal egg count and gastroscopy

A

b) Dental examination, rebreathing examination, faecal egg count, complete blood cell count and biochemistry analysis

143
Q

Which of the following antimicrobial drugs is effective when administered via the oral route in horses?
a) Penicillin
b) Amikacin
c) Gentamicin
d) Ceftiofur
e) Trimethoprim Sulphonamide

A

e) Trimethoprim Sulphonamide

144
Q

Which of the following dietary management protocols is most appropriate for treatment of equine metabolic syndrome?
a) Provide a diet primarly of long-stemmed fibre and provide carbohydrates at 15-20% of daily intake
b) Limit soluble carbohydrates and soak hay prior to feeding to limit the amount of carbohydrates ingested
c) Feed primarly lucerne hay but recommend a grazing muzzle to limit the amount of fresh grass consumed during pasture turn-out
d) Feed grass hay and allow no more than 50% of the day for grazing fresh grass in pasture

A

b) Limit soluble carbohydrates and soak hay prior to feeding to limit the amount of carbohydrates ingested

145
Q

A 3 month old foal is presented with pyrexia, cough and nasal discharge. The foal and its mother are housed in a dry lot with three other mares and their foals. Clinical examination of the foal shows increased respiratory rate and increased bronchovesicular sounds. Laboratory studies show leucocytosis with neutrophilia. Radiographs show a prominent alveolar pattern characterised by ill-defined regional consolidation. The most likely cause of these signs is infection with which of the following?
a) Rhodococcus equi
b) Equine rhinopneumonitis virus
c) Parascaris equorum
d) Equine influenza virus
e) Pneumocystis carinii

A

a) Rhodococcus equi

(3 month old foal)

146
Q

What is the most common inflammatory cell detected in the lower airways of a horse with moderate to severe equine asthma?
a) Neutrophil
b) Eosinophil
c) Mast cell
d) Lymphocyte

A

a) Neutrophil

147
Q

An endurance horse has just finished a 160km ride and is presented with colic signs and a high body temperature (40.8 degrees celsius). What is the preferred treatment?
a) Cooling
b) Enteral fluid therapy
c) Flunixin meglumine
d) Detomidine

A

a) Cooling

148
Q

At least what percentage of their body weight in forage (dry matter) does a horse need to consume daily in order to maintain appropriate gastrointestinal health?
a) 0.5%
b) 3%
c) 1%
d) None of the above - grain is more important than roughage

A

c) 1%

149
Q

Which of the following is the LEAST likely explanation for a positive bacterial culture from a tracheal wash taken via endoscopy from a horse with severe equine asthma?
a) Contamination of the sample due to non-aseptic technique
b) Primary bacterial infection
c) Normal upper respiratory tract flora
d) Opportunistic secondary bacterial infection

A

b) Primary bacterial infection

150
Q

Which diagnostic tool is most useful to confirm a definitive diagnosis of equine asthma?
a) Tracheal wash
b) Endoscopy
c) Bronchoalveolar lavage
d) Nasopharyngeal swab

A

c) Bronchoalveolar lavage

151
Q

An owner of a show jumping horse calls for some advice. She reports that the horse is rushing at fences and knocking down poles in competitions. He is reluctant to work in an outline and has been heard to make abnormal respiratory sounds when he pulls and his head and neck is in a more flexed position. He is also reported to cough occasionally at the start of exercise. What is the most appropriate advice?

A
  • Recommend conducting an endoscopic examination
  • Based on the owner’s reports ddx may include equine asthma, dynamic pharyngeal wall collapse etc.
  • Dynamic (exercise) endoscopy may also need to be conducted if no noticeable pathologies during resting endoscopy
152
Q

During a routine clinical examination of a 10 year old eventer you note an early grade 2 systolic murmur that does not extend to S2 and is localised over the heart base. Which of the following is the most appropriate response?
a) It is likely a murmur of mitral valve insufficiency; the horse should have a echocardiogram
b) It is likely a murmur of mitral valve insufficiency but is low grade; no action is required at this time
c) It is likely a murmur of tricuspid valve insufficiency but is low grade; no action is required at this time
d) It is most likely physiological flow murmur; no action is required
e) It is likely a murmur of aortic valve insufficiency; the horse should have an echocardiogram

A

e) It is likely a murmur of aortic valve insufficiency; the horse should have an echocardiogram

153
Q

Which of the following statements about gastrointestinal macromolecule absorption in the neonate is the most accurate?

A

Intestinal macromolecule absorption is greatest before 6hr age and reduces to zero by 24-36hr

154
Q

During a routine rectal examination in a horse, what will be palpable in the left dorsal and right dorsal abdominal quadrants respectively?
a) Pelvic flexure, left dorsal and ventral colon; caecum with two taenial bands
b) Loops of small intestine; caecum with two taenial bands
c) Spleen, kidney, nephrosplenic space; caecum with two taenial bands

A

c) Spleen, kidney, nephrosplenic space; caecum with two taenial bands

155
Q

During a rectal examination of a horse with mild to moderate colic you feel a structure with dough-like ingesta in the left-ventral abdominal quadrant extending towards the right. Choose the most likely diagnosis from the list below.
a) Pelvic flexure and left ventral colon impaction
b) Left dorsal displacement of the large colon
c) Right dorsal displacement of the large colon
d) Caecal impaction
e) Ileal impaction

A

d) Caecal impaction

156
Q

A variety of techniques should be used to ensure that a nasogastric tube is correctly placed within the oesophagus. Which of the following options is NOT a consistent method?

A
  • The horse will cough when you pass the tube into the trachea. Therefore, absence of a cough ensures correct placement.
157
Q

A 36-hr old foal has a history of not nursing and progressive weakness. On presentation, the foal is recumbent and dull. Physical exam reveals a rectal temperature of 36.8, heart rate of 140 bpm, RR of 38 bpm, congested mucous membranes with a CRT of 3 sec, serum lactate of 4 mmol/L and glucose of 4.2 mmol/L. CBC and biochemistry revealed a moderate leucopenia with a left shift neutrophilia and presence of cytoplasmic vacuolation of neutrophils as well as mild azotemia. What would be the most appropriate immediate plan for this foal?
a) Arterial blood gas, measure IgG concentrations, ultrasound abdomen, initiate enteral fluid therapy using mares’ milk and isotonic fluids, and treat with Na-penicillin and gentamicin
b) Blood culture, arterial blood gas, measure IgG concentrations, replace fluid deficits with Hartmann’s solution, start glucose CRI and treat with Na-penicillin and gentamicin
c) Blood culture, arterial blood gas, measure IgG concentrations, replace fluid deficits with Hartmann’s solution, start glucose CRI
d) Measure IgG concentrations, ultrasound abdomen, start on maintenance IV fluids using 0.45% NaCl and 5% dextrose

A

d) Measure IgG concentrations, ultrasound abdomen, start on maintenance IV fluids using 0.45% NaCl and 5% dextrose

158
Q

Which of the following is a key advantage of field exercise testing compared with treadmill exercise testing?
a) Environmental conditions can be easily controlled
b) Horses can be exercised to fatigue
c) Competition or racing conditions can be easily replicated
d) Respiratory measurements including breathing frequency, tidal volume and minute ventilation can be easily performed

A

c) Competition or racing conditions can be easily replicated

159
Q

Energy regulation involves a complex interaction of various body systems. Dysregulation of any of the regulatory systems in neonates commonly results in:
a) Hypothermia
b) Hypotension
c) Hypoglycaemia
d) Hypoproteinaemia

A

c) Hypoglycaemia

160
Q

During a rectal examination of a horse with severe colic you feel a severely distended colon just cranial to the pelvis, coursing in a horizontal direction. Choose the most likely diagnosis from the list below.
a) Pelvic flexure and left ventral colon impaction
b) Left dorsal displacement of the large colon
c) Right dorsal displacement of the large colon
d) Large colon volvulus
e) Caecal impaction

A

d) Large colon volvulus

161
Q

Oxygen insufflation in a neonate should be instituted when the PaO2 is:
a) <70 mmHg
b) <90 mmHg
c) <80mmHg
d) <60mmHg

A

d) <60mmHg

162
Q

The normal respiratory and heart rate range in a neonatal foal <12hr of age is:
a) 20-30 breaths per minute, 100-120 beats per minute
b) 40-50 breaths per minute, 80-100 beats per minute
c) 30-40 breaths per minute, 80-120 beats per minute
d) 10-20 breaths per minute, 60-80 beats per minute

A

b) 40-50 breaths per minute, 80-100 beats per minute

163
Q

Which of the following is NOT a cause of neonatal pneumonia?
a) Rhodococcus equi
b) Equine herpes virus 1
c) Chlamydia psittaci
d) Mixed bacterial population including anaerobes

A

a) Rhodococcus equi

164
Q

Which of the following cardiac findings are most likely to affect athletic performance negatively in a thoroughbred racehorse?
a) Grade 3/6 tricuspid regurgitation
b) Persistent atrial fibrillation
c) 2nd degree AV block at rest
d) Grade 2/6 mitral regurgitation

A

b) Persistent atrial fibrillation

165
Q

A 3 year old thoroughbred racehorse filly has just returned from an intense work out and is exhibiting signs of colic (pawing, biting at her sides). The filly is anxious and sweating. Palpation of the hindlimb musculature reveals a painful response. What is the most appropriate initial therapy indicated for this horse?
a) Cool the horse with iced water
b) NSAIDs administration IV
c) Buscopan administration IV
d) Fluid therapy IV or enteral

A

d) Fluid therapy IV or enteral

166
Q

Examine the endoscopic image taken from a racehorse during strenuous exercise. Which treatment option would be most appropriate in this horse?
a) “tie forward” procedure
b) Prosthetic laryngoplasty and ventriculocordectomy
c) Laser ventriculocordectomy
d) Partial arytenoidectomy

A

b) Prosthetic laryngoplasty and ventriculocordectomy

167
Q

Examine the following endoscopic image taken during exercise. What form of dynamic airway collapse is shown? (Hourglass shape of epiglottis)

A
  • Axial deviation of the aryepiglottal folds
168
Q

Which of the following forms of dynamic airway collapse is most likely to be associated with abnormal respiratory sounds?
- Axial deviation of the aryepiglottal folds
- Pharyngeal wall collapse
- Vocal fold collapse
- Dorsal displacement of the soft palate

A
  • Dorsal displacement of the soft palate
169
Q

From the endoscopic image below identify structure A
- Muscular process of the left arytenoid cartilage
- Muscular process of the right arytenoid cartilage
- Corniculate process of the left arytenoid cartilage
- Corniculate process of the right arytenoid cartilage

A
  • Corniculate process of the left arytenoid cartilage
170
Q

What is the most likely cause of a systolic murmur auscultated over the left apex and radiating dorsally?
tricuspid regurgitation
aortic regurgitation
flow murmur
mitral regurgitation

A

flow murmur

171
Q

Milk may be seen in the nares due to:
- Neonatal encephalopathy, sepsis, botulism
- Cleft palate, neonatal encephalopathy, botulism
- Neonatal encephalopathy, sepsis, pneumonia
- Cleft palate, choanal atresia, neonatal encephalopathy

A
  • Cleft palate, neonatal encephalopathy, botulism

Note: Neonatal encephalopathy and botulism may both cause dysphagia resulting in milk at the nares. A cleft palate results in communication between the oral and nasal cavities allowing milkflow into the
nasal passages and nares.

172
Q

Which of the following are NOT indications for resuscitation in a neonatal foal?
- The presence of euppnoea with a heart rate of <40bpm
- The presence of dusponea, apnoea or gasping
- A heart rate of <40bpm or > 50bpm but irregular
- The presence of eunoea with a heart rate of >60 bpm

A

?
Note: Eupnoea is the term used to describe normal breathing. CPR/resuscitation is indicated if a foal is: not responseive, not breathing (dyspnea/apnoea or gasping) or has a heart rate <40 bpm or <50bpm
and irregular/not inreasing/decreasing.

173
Q

Prematurity and dysmaturity have various characteristics. Choose the most accurate below.
- Low birth weight, small frame, shirt silky coat, floppy ears, domed head, poor thermal and glucose regulation.
- Low birth weight, small frame, long coat, flexural limb deformities, fully erupted incisors, poor thermal and flucose regulation

A
  • Low birth weight, small frame, shirt silky coat, floppy ears, domed head, poor thermal and glucose regulation.

Note: Characteritics of dysmature (or premature if a true dur date can truly be established based on the mares previous gestation lengths) foals include: low weight, small frame, flexural limb deformities
(usually laxity vs contracture), short fine, silky hair coat, domed forehead, floppy ears, soft rub cage, low lung compliance, incomplete ossification of cuboidal bones, muscle weakness (delayed nursing),
weak suckle, poor thermoregulation, glucose dysregulation, respiratory fialure, CV dysfunction (persistent hypotension), GI and renal dysfunction, entropion.

174
Q

Prematurity and dysmaturity have various aetiologies. Choose the most accurate list of causes.
- Placentitis, placental deparation, in-utero position, umbilical torsion, maternal systemic disease
- Plascentitis, placental separation, in-utero position, umbilical torsion, maternal over nutrition
- Placentitis, placental separation, twins, foals sepsis, maternal under nutrition, maternal systemic disease.
- Placentitis, placental separation, twins, umbilical torsions, maternal over-nutrition, maternal systmic disease.

A

?

175
Q

Neonatal encephalopathy is used ot describe various syndromes of neonatal neurological dysfunction. Which one of the following is not a cause of hypoxic-ishaemic encephalopathy/perinatal asphyxia
syndrome?
- Meconium aspiration
- Premature placental separation
- Dystocia
- Altered neurosteroid concentration

A

?

Note: Altered neurosteroid concentration are thought to be the cause of neonatal encephalopathy in foals that do not have evidence of/history of hypoxic or ishaemic episode. These foals are generally
descibred as/diagnosed with: neonatal encephalopathys –neonatal multisystemic maladjustment syndrome (NMMS).

176
Q

A foal is weak and lethargic 48hours after birth. The foal was normal at birth and began suckling vigourously at 1 hour of age. The dam’s last two foals died at 3 and 4 days of age, respectively. Examination
show icteric mucous membranes. LAboratory studies show hemoglobinuria. Which of the following is likely diagnostic?

  • Ion deficiency anaemia
  • Neonatal isoerythrolysis
  • Acute hepaticnecrosiis
  • Septicaemia
A
  • Neonatal isoerythrolysis

Note - Neonatal Isoerythrolysis (NI) = increased erythrocyte lysis. Foals with NI become progressively more anaemic with most cases being most severe at 3-4 days age. The other causes of icterus & haemoglobinuria are unlikely to recur in subsequent foals.

177
Q

Sepsis is defined as SIRS due to proven or suspected infection. Which of the following criteria is most correct when defining SIRS in neonates?
- Hypothemnia, tachycardia, tachyponea, leukopenia with a left shift neutrophilia, hyperlactamia and hypoglycaemia
- Hyperthermia, delayed skin tent, tachypnoea, leukocytosis with a lymphocytosis, hyperlactaemia and hypoglycaemia
- Hyperthermia, congested mucus membranes, delayed CRT, leukopenia with a left shift neutrophilia, hyperlactemia and hyperliemia

A
  • Hypothemnia, tachycardia, tachyponea, leukopenia with a left shift neutrophilia, hyperlactamia and hypoglycaemia

Note - SIRS criteria in neonates - require the presence of 3 or more of the following, 1 of which must be an abnormal temp or leukocyte count:
- Abnormal temp
- Increased HR
- Increased RR
- Increased / decreased WCC or >5% band neuts
- Increased lactate
- Decreased glucose

178
Q

A 5-day old foal has been diagnosed with Clostridium difficile assocaited diarrhoea. You replace her fluid deficits using Hartmann’s solution. Choosethe most correct list of ongoing treatments.
- Enteral nutrition with the mare’s milk, lactase supplementation, enteral isotonic fluids, probiotics, penicillin and gentamicin.
- IV NaCl 0.9% CRI, biosponge, probiotics, omeprazole, metronidazole, penicllin and gentamicin
- Enteral nutrition with mare’s milk, lactase supplementation, enteral isotinic fluids, biospone, metronidazole, penicillin and gentamicin.
- Enteral nutrition with mare’s milk, enteral isotonic fluids, lactase supplementation, biosponge, probiotics, probiotics, metronidazole, penicilling and gentamicin.

A
  • Enteral nutrition with mare’s milk, lactase supplementation, enteral isotinic fluids, biospone, metronidazole, penicillin and gentamicin.

Note - IV fluids may be required in addition to or in conjunction with the enteral fluids, although enteral isotonic fluids is often sufficient to maintain hydration in foals with less severe diarrhoea / gastrointestinal inflammation.

179
Q

A 2-day olf Thoroughbred colt present because the onwers noticed that he bas been straining and tail flagging. They hav enot seen him urinate since birth. After perfomring a full physical examination,
you perform a CBC and biochemistry which reveals mild azotaemia, and a moderate hyperkalemia (6.2mmol/L). Abdmonial ultrasound reveal increased free fluid within the peritoneal cavity.You conform
uroperitoneum due to a ruptured bladder. Choose the most appropirate next step:

  • Peritoneal drainage prior to surgical repair. Fluid therapy is not indicated.
  • Periotoneal drainage, IV fluid therapy with 0.45% NaCl and 5% dextrose prior to surgical repair.
  • Periotneal drainage and adminster Hartmann’s solution to replace fluid deficits prior to surgical repair.
  • Plan for emergency surgical repair.
A
  • Periotoneal drainage, IV fluid therapy with 0.45% NaCl and 5% dextrose prior to surgical repair.

Note - though most foals with uroperitoneum due to a ruptured bladder require surgical correction it is NOT a surgical emergency and the foal should always be stabilised prior to surgery by correcting fluid deficits and reducing whole body potassium through peritoneal drainage and glucose administration.

180
Q

Which of the following is not a cause of colic in neonatal foals?

  • Clostridium difficile assocaited diarrhoea
  • Parascaris spp. impaciton
  • Meconium impaction
  • Small intestinal intussusception
A
  • Parascaris spp. impaciton
181
Q

What is a common cause of colic in newborn foals?
- Atresia ani
- Meconium impaction
- Parascaris impaction
- Atresia coli

A
  • Meconium impaction
182
Q

What analgesia medication is most recommended to treat signs of colic in foals?

  • Meloxicam
  • Acepromazine
  • Detomidine
  • Xylazine
A
  • Meloxicam
183
Q

Enteritis is a common cause of Colic in foals?

  • Not true
  • Only true if associated with signs of diarrhoea
  • True
  • Only true if associated with signs of dehydration
A
  • True
184
Q

What are the most commoncauses of colic in neonatal foals?

  • Meconium obstipation, enteritis and intussusception
  • Meconium obstipation, enteritis and obstruction by Parascaris
  • Meconium obstipation, enteritis and necrotising enteritis
  • Meconium obstipation, enteritis and ruptured bladder
A
  • Meconium obstipation, enteritis and ruptured bladder
185
Q

The most common routine diagnostic tests in foals with colic signs are:

  • Rectal digital examination, abdominal ultrasound, abdominal radiographs and nasogastric intubation
  • Rectal digital examination, abdominal radiographs, blood analysis and proctoscopy
  • Abdominal radiographs, abdominal ultrasound, gastroscopy and abdominocentesis
  • Gastroscopy, blood analysis, abdominocentesis and nasogastric intubation
A
  • Rectal digital examination, abdominal ultrasound, abdominal radiographs and nasogastric intubation
186
Q

Which of the following treatments is most important for a horse demonstrating signs of severe hepatoencephalopathy (i.e. head-pressing, intermittent maniacal behavior, staggering, etc.)?

  • Broad-spectrum antibiotic therapy
  • Use of benzodiazepine sedative
  • Use of an alpha-2 agonist sedative
  • Administration of lactulose and vinegar
A
  • Use of an alpha-2 agonist sedative
187
Q

A 3 week-old foal is presented with acute signs of severe depression, tachycardia, tachypnea, hyperthermia, and slightly icteric mucosal membranes. Abnormal laboratory findings include: hypoglycemia,
marked elevation of liver enzymes in the serum and a very high conjugated bilirubin concentration. What is the most likely diagnosis?

  • Chronic active hepatitis caused by pyrrolizidine alkaloid intoxication
  • Tyzzer’s disease caused by clostridium piliforme
  • Choledocholelithiasis caused by ascending enteric bacteria like E. coli
  • Theiler’s disease caused by the use of tetanus antitoxin in the mare
A
  • Tyzzer’s disease caused by clostridium piliforme
188
Q

What are the classical histopathology findings of a liver biopsy of horses with pyrrolizidine alkaloid toxicity.

  • Megalocytosis, periportal fibrosis and bile duct hyperplasia
  • Megalocytosis, bile duct fibrosis & hyperplasia
  • Periportal fibrosis, bile stoneand bile duct hyperplasia
  • Lymphocytes & neutrophils, periportal fibrosis and bile duct hyperplasia
A
  • Megalocytosis, periportal fibrosis and bile duct hyperplasia
189
Q

Describe your diagnostic plan for a horse with liver disease

  • Specific & non-specific liver enzyme activities, liver ultrasound, and liver biopsy.
  • Specific liver enzyme activities, liver palpation, and liver biopsy.
  • Specific & non-specific liver enzyme activities, liver ultrasound, and blood ammonia concentration
  • Liver function tests, liver ultrasound, and liver biopsy.
A
  • Specific & non-specific liver enzyme activities, liver ultrasound, and liver biopsy.
190
Q

Which of the following tests do you consider important during your diagnostic testing in a horse suspected of liver disease.

  • AST, ALP, bile acids and ammonia
  • AST, ALP, bilirubine and ammonia
  • GSDH, GGT, bile acids and bilirubine
  • CK, GGT, bile acids and bilirubine
A
  • GSDH, GGT, bile acids and bilirubine
191
Q

Laryngeal ultrasonogrpahy is a useful diagnostic aid in which of the following conditions?

  • Intermittent dorsal displacement of the soft palate
  • Pharyngeal wall collapse
  • Epiglottal entrapment
  • Arytenoid chondritis
A
  • Arytenoid chondritis

Note - laryngeal ultrasound enables the arytenoid cartilage to be examined more thoroughly than endoscopy and to identify thickening of the cartilage

192
Q

A 7 year old warmblood gelding used for advanced level dressage is present with a very loud respiratory noise wihtout exercise intolerance. Grade III.3 laryngeal dysfunction I diagnosed during resting
endoscopic examination. What treatment is recommended?

  • A protestheic laryngoplasty (‘tie back’) performed on the horse standing under sedation
  • A standing bilateral trans-endoscopic ventriculo-cordectomy using a diode laser
  • A 6 week course of oral dexamethasone with dust management of the environment
  • A prosthetic laryngoplasty (‘tie back’) performed on the horse under general anaesthesia
A
  • A standing bilateral trans-endoscopic ventriculo-cordectomy using a diode laser
193
Q

Which of the following best describes the advantage of field anaesthesia compared to standing sedation?

  • It allows for better patient immobility
  • Cardiorespiratory function is maintained better
  • It is more appropriate for longer procedures
  • It requires less personnel to safely implement
A
  • It allows for better patient immobility
194
Q

A well trained, easily handled and healthy mare is present for a Caslick’s procedure. Which of the following sedation protocols is the most appropriate choice for this mare?

  • Xylazine and application of a twitch
  • Midazolam and application of twitch
  • Detomidine followed by butorphanol
  • Acepromazine, followed by butorphanol
A
  • Detomidine followed by butorphanol
195
Q

Which of the following is generally not considered to be a complication associated with general anaesthesia in a horse?

  • Hypoxaemia
  • Hypotension
  • Tachycardia
  • Myopathy
A
  • Tachycardia
196
Q

An 18 year old Arabian horse is suspected of having aortic regurgitation because of which of the following findings during heart auscultation?

  • Holosystolic decrescendo cardiac murmur on the left hemithorax
  • Mid-end diastolic squeaking cardiac murmur on the left hemithorax
  • Pansysthoutolic cardiac murmur on the left hemithorax
  • Holosystolic crescendo cardiac murmur on the left hemithorax
A
  • Holosystolic decrescendo cardiac murmur on the left hemithorax
197
Q

In horses with chronic Streptococcus equi subsp equi infection, what is the most appropriate diagnostic technique to positively identify the aetiologic agent?

  • Sterile blood collection and blood culture
  • Nasal swab and culture
  • Guttural pouch lavage and PCR testing
  • Transtracheal wash and culture
A
  • Guttural pouch lavage and PCR testing
198
Q

Choose the most appropriate diagnosis reading equine herpes virus (EHV) infection in horses:

  • Vaccination is commonly performed and is highly protective for the neurological form of EHV-1
  • EHV-1 causes respiratory illness and ultimately targets the epithelium
  • Virus isolation and PCR of nasopharyngeal swabs may be used to support a diagnosis of EHV-1
  • EHV-1 is transmitted by infected fomites, vertical transmission and contaminated surgical instruments
A
  • Virus isolation and PCR of nasopharyngeal swabs may be used to support a diagnosis of EHV-1
199
Q

Based on the bacteria most commonly associated with bronchopneumonia in adult horses, which of the following provides the most appropriate antimicrobial regiment?

  • Enrofloxacin, gentamicin and Polymixin B
  • Trimethoprim sulphate and ceftiofur
  • Penicillin, gentamicin and metronidazole
  • Ceftiofur, chloramphenicol and metronidazole
A
  • Penicillin, gentamicin and metronidazole
200
Q

A horse is presented in severe respiratory distress and a history of fever, recent travelling, general anaesthesia or previous illness, what would be the most logical and appropriate course of action from the
following?

  • Sedate the horse and perform emergency tracheostomy, administration of intranasal oxygen, broad-spectrum antimicrobials and bronchodilators
  • Perform an endoscopy to obtain a tracheal lavage for culture and susceptibility, administration of broad-spectrum antimicrobials, bronchodilators and NSAIDs
  • Sedate the horse to perform emergency tracheostomy, administration of broad-spectrum antimicrobials, inhaled corticosteroids and laminitis prevention
  • Perform thoracocentesis to drain pleural fluid, administration of broad-spectrum antimicrobials, NSAIDs and laminitis prevention
A
  • Perform thoracocentesis to drain pleural fluid, administration of broad-spectrum antimicrobials, NSAIDs and laminitis prevention
201
Q

A horse is presented for repair of a 5cm laceration over his left eye but the horse is reluctant to allow IV access. Which of the following protocols is most appropriate in this scenario?

  • Administer xylazine IM, top-up IV if necessary and perform local blocks
  • Dispense oral acepromazine, reschedule to later in the day when the horse is sedated
  • Administer midazolam IM, administer medetomidine IV once the horse is sedated and followed by methadone IV
  • Apply a twitch, administer midazolam IM and perform local blocks
A
  • Administer xylazine IM, top-up IV if necessary and perform local blocks
202
Q

A 7 year old Clydesdale cross gelding is presented with roaring inspiratory sounds occasional cough and exercise intolerance. He performs low level carriage driving. His resting endoscopy shows a III.3
laryngeal function. What would be the most appropriate approach for this horse?

  • Rest the horse for 6 weeks with a course of oral prednisolone
  • Refer the horse to perform a bilateral laser ventriculo-cordectomy
  • Refer the horse to perform a laser ventriculo-cordectomy and laryngeal tie-back procedure
  • Perform a BAL and rule out lower airway inflammatory disease
A
  • Perform a BAL and rule out lower airway inflammatory disease
203
Q

Which is an important consideration when using ketamine top ups during a castration performed in the field?

  • Active metabolite will build up in circulation
  • Ketamine is more expensive than guaifenesis
  • Apnea will develop
  • Hypotension will develop
A
  • Active metabolite will build up in circulation
204
Q

Which clinical signs and diagnostic procedures confirm an equine herpes virus infection?

  • Horses with nasal discharge, cough and increased antibody in serum titre or serology
  • Horses with nasal discharge, severe cough and virus isolation from nasal swab
  • Horses with a cough, fever and PCR from a guttural pouch lavage
  • Horses with nasal discharge, fever and positive PCR from a nasopharyngeal swab
A
  • Horses with nasal discharge, fever and positive PCR from a nasopharyngeal swab
205
Q

What is the most appropriate treatment regime for a horse with recurrent airway obstruction (RAO)?

a. low dust environment, bronchodilator and corticosteroids
b. low dust environment, bronchodilator and non-steroidal anti-inflammatory
c. low dust environment, antibiotics and corticosteroids
d. low dust environment, antibiotics and bronchodilator

A

a. low dust environment, bronchodilator and corticosteroids

206
Q

How can RAO be best distinguished from inflammatory airway disease during the clinical examination?
a. Based on bronchoalveolar lavage findings. Horses with RAO will have a higher proportion of neutrophils
b. Horses with RAO will cough, whilst horses with IAD will not
c. Horses with RAO will have evidence of increased respiratory effort at rest, whilst horses with IAD will not
d. Based on tracheal wash findings. Horses with RAO will have a higher proportion of neutrophils

A

c. Horses with RAO will have evidence of increased respiratory effort at rest, whilst horses with IAD will not

207
Q

Which of the following diagnostic tools is most useful to confirm a definitive diagnosis of lower airway inflammation?
a. Tracheal wash
b. Bronchoalveolar lavage
c. Tracheal endoscopy
d. Nasopharyngeal swab

A

b. Bronchoalveolar lavage

208
Q

What is the most likely diagnosis for a horse that is presented with bilateral purulent nasal discharge and fever without a cough?
a. Strangles disease
b. Pleuropneumonia
c. Recurrent Airway Obstruction (RAO)
d. Bronchopneumonia

A

a. Strangles disease

209
Q

A 2-year old horse develops acute swelling of the head two weeks after drainage of a submandibular abscess. Your tentative diagnosis is purpura haemorrhagica. What are the two most important
components of the treatment of equine purpura haemorrhagica?
a. Immunosuppressive corticosteroid treatment (IV or IM) and IV fluids
b. IV fluids and antimicrobial treatment (procaine penicillin IM)
c. Immunosuppressive corticosteroid treatment (IV or IM) and diuretics (furosemide IV)
d. Immunosuppressive corticosteroid treatment (IV or IM) and antimicrobial treatment (procaine penicillin IM)

A

d. Immunosuppressive corticosteroid treatment (IV or IM) and antimicrobial treatment (procaine penicillin IM)

210
Q

Which of the following diagnostic techniques is the most appropriate for making a definitive diagnosis of dynamic upper airway collapse in the horse?
a. Sound analysis
b. Exercising endoscopy
c. Resting endoscopy
d. Laryngeal ultrasound

A

b. Exercising endoscopy

211
Q

Which of the following statements best describes the situation in a normal horse during exercise?
a. Soft, blowing sounds are normal during expiration but not during inspiration
b. Loud respiratory noises are audible throughout the respiratory cycle
c. There should be no audible respiratory sounds during inspiration or expiration
d. Soft, blowing sounds are audible during inspiration but not during expiration

A

a. Soft, blowing sounds are normal during expiration but not during inspiration

212
Q

A deficiency of which of the following electrolytes is a possible risk factor for development of atrial fibrillation?
a. Sodium
b. Calcium
c. Potassium
d. Chloride

A

c. Potassium

213
Q

You diagnose atrial fibrillation in a Dutch warmblood dressage horse. It was previously auscultated one month ago and found to be in normal sinus rhythm with no murmurs. What are the chances of
successfullyconverting this horse to sinus rhythm using quinidine sulphate and what is the likelihood of recurrence?
a. 25% chance of conversion, 95% chance of recurrence
b. 95% chance of conversion, 65% chance of recurrence
c. 95% chance of conversion, 25% chance of recurrence
d. 65% chance of conversion, 95% chance of recurrence

A

c. 95% chance of conversion, 25% chance of recurrence

214
Q

Which of the following is likely to be a common side effect following the administration of quinidine for treatment of atrial fibrillation?
a. Tachycardia
b. Colitis
c. Laminitis
d. Bradycardia

A

a. Tachycardia

215
Q

Although there are many acceptable techniques for use in field anaesthesia, there are core principles that should be completed in order for its safe performance. Which of the following best describes one
of these principles?

  • Intravenous catheters should be placed prior to induction to ensure immediate venous access for drug delivery
  • Monitoring during field anaesthesia is vital and should involve the use of a blood pressure monitor
  • Owners must be fully informed about all stages of the procedure in order to help on the horses head for induction
  • A horse can be best assisted during recovery by a person with a lead rope attached to a sturdy halter
A
  • Intravenous catheters should be placed prior to induction to ensure immediate venous access for drug delivery
216
Q

Recovery from general anaesthesia in horses can be challenging and with a high incidence of complications. Which is not a common complication seen in recovery of horses?
- Hypotension
- Colic
- Facial nerve paralysis
- Quadriceps myopathy

A
  • Hypotension (seen during anaesthesia not recovery)
217
Q

‘Mad galloper is a 3-year-old thoroughbred gelding who is working at a very high level, in preparation for a major race. There is no history of poor performance and you are visiting the farm to vaccinate the
horse. On physical exam you notice some missing beats on what sounds like an otherwise sinus rhythm. You take an ECG to characterise the problem and the results are below. HR: 28 bpm. Please select
what your conclusion should be.

A
  • It does not worry me because this is a very common finding in horses, it is due to their high vagal tone, especially in fit horses (would only be worried if there was two in a row)
218
Q

Which of the following propositions is correct?

  • Guttural pouches are crossed by branches of the cranial nerves IX and XIII in the lateral compartment
  • On x-rays, guttural pouches appear as radiolucent areas at the back of the skull and in front of the first and second cervical vertebrae in normal horses.
  • Guttural pouches are paired and divided by the basi-hyoid bone into two unequal compartments
  • A large segment of the stylohyoid bone divides each guttural pouch in one small lateral compartment and one large medial compartment
A
  • A large segment of the stylohyoid bone divides each guttural pouch in one small lateral compartment and one large medial compartment
219
Q

Choose the most appropriate answer regard equine influenza virus (EIV) in horses:

  • Vaccination is not commonly performed due to antigenic drift
  • Antigenic drift is cause by a mutation in haemagglutinin
  • EIV is a single stranded DNA virus displaying antigenic drift
  • EIH has a high mortality but low morbidity and is commonly identified in outbreaks of respiratory disease
A
  • Antigenic drift is cause by a mutation in haemagglutinin
220
Q

Which of the following options in the most appropriate for the treatment of a 3cm x 3cm progressive ethmoid hematoma lesion?
- Mass removal via maxillary bone flap
- Cryosurgery via fronto-nasal bone flap
- Chemical ablation using formalin injection
- Endoscopy guided transection and removal

A
  • Chemical ablation using formalin injection

Note - mass removal is possible however the lesion is small and you can just use chemical ablation. You may need to use multiple injections –however, it is less invasiveand should be tried first. IF the lesion was
larger then you would try mass removal or cryosurgery.

221
Q

Which is the safest protocol for providing enteral nutrition to a 30kg dysmature foal kg?
- 125mL of warm mare’s milk via NG tube q2 h
- 500mL of warm mare’s milk via NG tube q 2 h
- 250mL or warm mare’s milk + 5mL honey via NG tube q 2 h
- 375mL of warm mare’s milk via NG tube q 6 h

A
  • 125mL of warm mare’s milk via NG tube q2 h
222
Q

Select the best answer for possible causes for failure of passive transfer on a foal?

  • Lack of colostrum intake before 12 hours of age, poor colostrum quality, premature lactation on mare, agalactic/dysgalactica, mare dies, mare rejects foal, foal is unable to nurse due to disease.
  • Lack of colostrum intake before 3 hours of age, mare did not like her foal, pyrrolizidine alkaloid toxicity on the mare, foal doesn’t know where to nurse from.
  • Lack of colostrum intake before 4 hours of age, mare doesn’t wax up before foaling, mare retained her placenta, gossypol toxicity on the mare, mare dies, foal is sick and cannot nurse.
  • Lack of colostrum intake before 12 hours of age, poor colostrum quality, agalactic/dysgalactica, hormonal imbalances on the mare, cleft palate, dummy foal.
A
  • Lack of colostrum intake before 12 hours of age, poor colostrum quality, premature lactation on mare, agalactic/dysgalactica, mare dies, mare rejects foal, foal is unable to nurse due to disease.
223
Q

You have a 2-day old, 30kg filly with maladjustment syndrome and you are suspicious about immature GI and pharyngeal dysfunction because you notice milk coming from her nostrils. She is otherwise
healthy. You pass an indwelling NG tube to provide enteral nutrition. Select the best answer the describes how you would feed her every 2 hours with milk, starting at 6% of her body weight?

  • Check for reflux. Milk needs to be warm, filly needs to be in sternal or standing. Feed her 150mL of milk q 2 hours and flushthe tube with water.
  • Check for reflux. Milk need to be warm, filly can be laying down or in sternal. Feed her 300mL of milk q 2h and flush the tube with water.
  • Check for reflux. Milk can be at any temperature. Filly needs to be in sternal or standing. Feed her 200mL of milk q 2 hours and chase the filly afterwards.
  • Check for reflux. Milk can be at room temperature. Filly needs to be standing. Feed her 50mL of milk q 2 hours and lay her down immediately.
A
  • Check for reflux. Milk needs to be warm, filly needs to be in sternal or standing. Feed her 150mL of milk q 2 hours and flushthe tube with water.
224
Q

A two year old thoroughbred mare present to you for fever and poor appetite one day duration. The mare has history of coughing and bilateral serous nasal discharge. What is the most appropriate plan of
action?

  • Further diagnostics are recommended to better characterise the condition, such as a complete physical exam, rebreathing exam,complete blood cell count and fibrinogen concentration.
  • The mare likely has a bacterial pneumonia and I am going to treat her with a course of IM procaine penicillin, because potentiated sulphonamides are usually not effective.
  • The mare likely has a viral respiratory infection and the use of antimicrobials is unnecessary.
  • Treating the mare with antimicrobials is best. It will treat the bacterial pneumonia, or prevent the more from developing any seconday bacterial pneumonia in case of a viral infection.
A
  • Further diagnostics are recommended to better characterise the condition, such as a complete physical exam, rebreathing exam,complete blood cell count and fibrinogen concentration.
225
Q

Horses with ventricular septal defect have a:
- Systolic murmur on the right side, point of maximal intensity (PMI) in the apex area
- Systolic murmur on the left side, point of maximal intensity (PMI) in the apex are
- Diastolic murmur on the left side, point of maximal intensity (PMI) in the apex area
- Diastolic murmur on the right side, point of maximal intensity (PMI) in the apex area

A
  • Systolic murmur on the right side, point of maximal intensity (PMI) in the apex area
226
Q

A 4-year-old thoroughbred racehorse is presented with a history of poor performance and loud respiratory sounds during inspiration. At rest, his laryngeal function is graded III.I on the Havemeyer scale.
The following image is recorded during exercising endoscopy.

  • Recurrent laryngeal neuropathy
  • 4th branchial arch defect
  • Axial deviation or aryepiglottal folds
  • Arytenoid chondritis
A
  • Recurrent laryngeal neuropathy

Note - Answer is recurrent laryngeal neuropathy. Left side is affected, obstructing the rima glottis. Quite pathognomonic. 4th brachial arch defect – mainly on the right side
Axial deviation of the aryepiglottic fold – you will still see a straight line of the cartilage, but you will see an hour-glass shape
Arytenoid chondritis –can’t really see them, only see sign of inflammation (can potentially see pus draining

227
Q

Which of the following treatment option is the least appropriate for the treatment of a mild small colon faecal impaction?
- Surgery: placement of rectal liner
- IV and oral rehydration
- Antimicrobials +/- polymixin B
- Laxatives such as mineral oil, MgSo4 and DSS

A
  • Surgery: placement of rectal liner
228
Q

Which of the following would preclude you from administering enteral therapy for the management of colic?

  • Lack of faecal output for the past 24 hours
  • Administration of enteral water 2 hours previously
  • 4L of nasogastric reflux
  • Reduced intestinal borborygmi on auscultation
A
  • 4L of nasogastric reflux

Note - 4L of reflux is a lot. Can give enteral fluids every 2 hours -as long as there’s no reflux, this route is preferred. Reduction of borborygmic can be due to colic or reduce feed intake.

229
Q

You are being presented with a 5-year-old Arabian gelding showing signs of mild intermittent abdominal pain two days after being discharged from hospital for a routine arthroscopy. Key findings on the
colic workup are: decreased faecal outpit and a tight taenia band on the right side on rectal examination. Which of the following conditions is most likely cause of the abdominal band?

  • Caecal tympany
  • Caecal impaction
  • Caecocaecal or cecocolicintussusception
  • Large colon displacement
A
  • Caecal impaction

Note - Right sided will be something with the caecum. Intussusception is not common. Impaction –after surgery is quite common. Answer is caecal impaction.

230
Q

You are examining a 12yo gelding, which presents with moderate signs of colic, loose faeces and signs of depression. You note a deep purple MM and a CRT of 3.5 seconds on the physical examination.
On US exam of the right dorsal abdomen you measure an intestinal wall thicknessof 9mm. You understand the severity of the situation and consider your treatment options. Which of the following should not
be part of your considerations?

  • Look for antimicrobials sparing the lipid A region of the endotoxin
  • Prevent synthesis, release or action of inflammation mediators
  • Assess severity of SIRS – systemic Inflammatory response syndrome
  • Neutralizeany circulating endotoxin molecules and aggregates
A
  • Look for antimicrobials sparing the lipid A region of the endotoxin

Note: Answer is A becauseyou are using polymyxin B to bind to the region of the toxin so don’t have to worry about it

231
Q

Which of the following is true (one answer only):
- Horses have brachydont teeth which have a crown above the gingival line with pointed occlusal surface good for tearing and shredding
- Horses have hypsodont teeth which have a crown above the gingival line with pointed occlusal surface good for tearing and shredding.
- Horses have brachydont teeth which are high crowned teeth with rough and flat occlusal surface good for crushing and grinding.
- Horses have hypsodont teeth which are high crowned teeth with a rough and flat occlusal surface good for crushing and grinding

A
  • Horses have hypsodont teeth which are high crowned teeth with a rough and flat occlusal surface good for crushing and grinding
232
Q

In regard to the treatment recommendations for Equine Glandular Gastric Disease:

  • The addition of sucralfate to omeprazole treatment is recommended and these ulcers take longer to heal than lesions in the squamous mucosa
  • The addition of sucralfate to omeprazole treatment is not recommended and these ulcers take longer to heal than lesions in the squamous mucosa
  • The additionof sucralfate or omeprazole treatment is recommended and theses ulcers heal faster than lesions in the squamous mucosa.
  • The addition of sucralfate to omeprazole treatment is not recommend and these ulcers heal faster than lesions in the squamous mucosa.
A
  • The addition of sucralfate to omeprazole treatment is recommended and these ulcers take longer to heal than lesions in the squamous mucosa
233
Q

Which is the most consistent laboratory finding or right dorsal colitis?
- Hypoalbuminaemia
- Azotaemia (increased plasma creatinine concentration)
- Faecal occult blood
- Neutropenia

A
  • Hypoalbuminaemia
234
Q

Which enzyme (measure in blood) is most specific for hepatopathy in horses?
- GGT – Gamma-glutamyl transferase
- LDG –Lactate dehydrogenase
- AST - Aspartate amino transferase
- ALP – Alkaline phosphatase

A
  • GGT – Gamma-glutamyl transferase

Note - Sometimes when you have an impaction/displacement, if you have a lot of weight on the liver you can also cause some elevation of GGT.

235
Q

The diagnosis salmonellosis and clostridiosis in adult horses differ in what way?

  • Horses with salmonellosis are treated with antimicrobials but those with clostridiosis do not
  • Diagnosis of salmonellosis requires PCR of the faecal sample, whereas diagnosis of clostridiosis requires detecting faecal organisms
  • Diagnosis of salmonellosis requires repeated faecal bacterial culture whereas diagnosis of clostridiosis requires detecting faecal organisms and toxins
  • Horses with clostridiosis exhibit fever, laminitis but those with salmonellosis do not
A
  • Diagnosis of salmonellosis requires repeated faecal bacterial culture whereas diagnosis of clostridiosis requires detecting faecal organisms and toxins
236
Q

Please indicate the correct statement about peritonitis:

  • Peritoneal adhesions commonly form after severe peritonitis and are associated with excessive fibrin formation and/or deficient fibrinolysis.
  • The use of blunt cannula (e.g. teat cannula) for abdominocentesis eliminates the risk of intestinal penetration and peritoneal contamination with intestinal contents.
  • Actinobacillus equuli is a relatively common cause of septic peritonitis typically causes severe disease with high fatality rate (90-100%)even when aggressive treatment is given.
  • Abdominal surgery only leads to peritonitis if there is significant peritoneal contamination and/or excessive physical or chemical aggression to the peritoneum.
A
  • Peritoneal adhesions commonly form after severe peritonitis and are associated with excessive fibrin formation and/or deficient fibrinolysis.

Note: Blunt cannula can be atraumatic but its not really a gold standard. If you have sand impaction it actually not great to use. Actinobacillus – fatal rate not that high (can be dealt with using aggressive
tx). Any contamination can lead to peritonitis. Serosa can get adhesion when you cause any trauma, that’s why you use belly jelly.

237
Q

Please indicate the correct statement about rectal tears during transrectal palpation:
- Regardless of tear severity (grade), antibiotics are indicated.
- In all cases, this outcome should be attributed to veterinary error.
- Regardless of tear severity (grade), the horse needs to be transported to a referral centre.
- If the peritoneal cavity has been contaminated, the options are immediate euthanasia or aggressive treatment.

A
  • If the peritoneal cavity has been contaminated, the options are immediate euthanasia or aggressive treatment.

Note - If you have a great 4 rectal tear, all the way into the abdomen –severe contamination requires immediate euthanasia. Just going through the mucosa you don’t really need Abs - weigh up the risk and benefit.
Grade 1 or 2 you don’t have to refer.

238
Q

A 500kg horse with 5% dehydration and about 5 L of enterogastric reflux (drained through the nasogastric tube) every 4 hours. Considering the rate of enterogastric reflux to be constant and the daily
requirement of water for maintenance to be 50ml/kg/day, please indicate the minimum volume of fluids to be infused during the first 24h with the aim of preventing further dehydration:
- 55L
- 80L
- 60L
- 65L

A
  • 80L

Fluid deficit is 5% x 500 = 25L. Ongoing loss is 30L. Maintenance is 25L.

239
Q

Please indicate the correct statement about diseases of the small intestine:
- Epiploic foramen entrapment usually leads to strangulating obstruction of a segment of jejunum and/or ileum.
- Duodenojejunal intussusception is the most common type of intussusception in the horse.
- Ilieal impaction can only be treated surgically.
- Inguinoscrotal hernias are more common in horses that have the habit of cribbing.

A
  • Epiploic foramen entrapment usually leads to strangulating obstruction of a segment of jejunum and/or ileum.

Note: Cribbing is associated in epiploic foramen entrapment. You can treat impaction medically FIRST. Duodenojejuno intussusception is almost impossible, jejuno-jejunal is more common.

240
Q

The image below is a necropsy photo in a horse that exhibited fever 40.5oC, depression, anorexia, leukopaenia, increased protein (70g/L) in the peritoneal fluid. The horse was euthanized since the owner was not willing to invest more than $1000. What can be said about this case if the budget was limited.

  • If this horse is treated with potassium penicillin, gentamicin and metronidazole, it would likely be possible to discontinue antibiotic therapy after 5 days.
  • This is likely a case of non-septic peritonitis because the peritoneal fluid is not purulent.
  • A drain placed in the ventral abdomen for peritoneal lavage would likely be an effective route for rapid ‘cleaning; of the peritoneal cavity.
  • If the peritonitis response to nonsurgical treatment, this horse is still likely to develop peritoneal adhesions.
A
  • If the peritonitis response to nonsurgical treatment, this horse is still likely to develop peritoneal adhesions.

Note: Cannot be easily fixed, you already have extensive fibrin build up, if the horse would have survived then 100% adhesion will developed.

241
Q

The image below is a photo taken during colic surgery. Please indicate the affected segment of the GI tract, the primary problem, the type of horse predisposed to this condition and a complication
commonly seen before surgery:

  • Small intestine, lipoma, Arabian, enterogastric reflux
  • Small colon, lipoma, Thoroughbred, enterogastric reflux
  • Small colon, impaction, miniature horses, tympany
  • Small intestine, Ascarid impaction, Arabian, tympany
A
  • Small colon, impaction, miniature horses, tympany

Note: Small intestine only have 1 band, colon have 2 bands. Lesion is not a lipoma because you can see a hard object impacting the colon. Lead to tympany because gas built up.

242
Q

Please indicate the correct statement about oesophageal impaction (choke):
- At least initially, more commonly, the obstruction is located in the cervical oesophagus.
- Chose is typically a recurrent problem and most horses have to be euthanatized.
- Poor dentition, oesophageal stenosis, slow feed intake and ingestion of a slurry are risk factors for chose.
- Pushing the impaction, the cervical oesophagus into the thoracic oesophagus with a tube is indicated because it is easier to operate the thoracic oesophagus than the cervical oesophagus.

A
  • At least initially, more commonly, the obstruction is located in the cervical oesophagus.

Note: Correct answer is the first option, most of the presenting case is around the cervical area. Can recurrent because stricture formation – but can be managed don’t have to euthanasia. Poor dentition and
stenosis can lead to choke. Slow feed intake and slurry is actually what you want. Last option is self-explanatory à you don’t want to do that.

243
Q

What is the most common type of inflammatory cell response identified in bronchoalveolar lavage fluid in Australian horses with mild to moderate asthma?
- Lymphocytic response
- Neutrophilic response
- Eosinophilic response
- Mixed inflammatory cell response

A
  • Mixed inflammatory cell response
244
Q

The most common cause of icterus in adult horses are:
- Haemolysis, liver disease and anorexia
- Disseminated intravascular coagulation (DIC), blood loss anaemia and haemolysis
- Sepsis, hyperlipaemia and anorexia
- Hyperbilirubinia, phototoxins and lymphoma

A
  • Haemolysis, liver disease and anorexia
245
Q

A newborn foal is presented with a patent urachus which developed 4 days after birth. Which of the following is an indicator for surgical ablation of the umbilical remnants?
- The foal is a male and the urachus is unlikely to close spontaneously
- The foal has a leukocytosis and elevated serum amyloid A (SAA) concentration
- The ultrasound examination of the umbilicus revealed an abnormal enlargement of the umbilical vein
- During urination more urine is passed from the urachus than the urethra

A
  • The ultrasound examination of the umbilicus revealed an abnormal enlargement of the umbilical vein
246
Q

Which of the following approaches to the abdomen is most commonly used for exploratory colic surgery?

A
  • Ventral midline incision
247
Q

Which of the following GI segments CANNOT be exteriorised but palpated when performing a ventral midline exploratory celiotomy?

A
  • Duodenum
248
Q
A